Download as pdf or txt
Download as pdf or txt
You are on page 1of 71

1.

Which of the following conditions usually causes hypokalaemia in the absence of


hypertension?ia

A.A Cushing's syndromeia


B.A Conn's syndromeia
C.A 11-beta hydroxylase deficiencyia
x D.A Bartter's syndrome
E.A Liddle's syndromeia
Next question
Bartter's syndrome is associated with normotension
Hypokalaemia and hypertension
sqweqwesf erwrewfsdfs adasd dhe
For the MRCP it is useful to be able to classify the causes of hypokalaemia in to those
associated with hypertension, and those which are not
he earaer aeraer asdsadas eerw dssdfsselleds
Hypokalaemia with hypertension
• Cushing's syndromehe
• Conn's syndrome (primary hyperaldosteronism)he
• Liddle's syndromehe
• congenital adrenal hyperplasia (11-beta hydroxylase deficiency)he

he earaer aeraer asdsadas eerw dssdfsselleds


Carbenoxolone, an anti-ulcer drug, and liquorice excess can potentially cause
hypokalaemia associated with hypertension
he earaer aeraer asdsadas eerw dssdfsselleds
Hypokalaemia without hypertension
• diureticshe

• GI loss (e.g. diarrhoea, vomiting)he


• renal tubular acidosis (type 1 and 2*)he
• Bartter's syndromehe
• Gitelman syndromehe
he earaer aeraer asdsadas eerw dssdfsselleds
Urinary potassium may be helpful in diagnosis, if < 30mmol/day then GI loss or diuretic
use is likely to be the cause
he earaer aeraer asdsadas eerw dssdfsselleds
*type 4 renal tubular acidosis is associated with hyperkalaemia
2.A 7-year-old boy is being investigated for failure to thrive and generalised weakness.
His blood pressure is normal. The following blood results are obtained:
he earaer aeraer asdsadas eerw dssdfsselleds
Na+ 137 mmol/l
K+ 3.0 mmol/l
Urea 4.5 mmol/l
Creatinine 65 µmol/l
Bicarbonate 33 mmol/l
eraer asdsadas eerw dssdfsselleds
What is the most likely diagnosis?ia

A.A Conn's syndromeia


x B.A Bartter's syndrome
C.A Cushing's syndromeia
D.A 21-hydroxylase deficiencyia
E.A Liddle's syndromeia
Next question
Bartter's syndrome is associated with normotension
Bartter's syndrome is the most likely diagnosis. Congenital adrenal hyperplasia due to 21-
hydroxylase deficiency is associated with precocious puberty rather than failure to thrive
in boys. Both Conn's and Cushing's are associated with hypertension and are not common
in this age group
Bartter's syndrome
sqweqwesf erwrewfsdfs adasd dhe
Bartter's syndrome is an inherited cause (usually autosomal recessive) of severe
+ +
hypokalaemia due to defective chloride absorption at the Na K 2Cl- cotransporter in the
ascending loop of Henle. It should be noted that is associated with normotension (unlike
other endocrine causes of hypokalaemia such as Conn's, Cushing's and Liddle's syndrome
which are associated with hypertension)
he earaer aeraer asdsadas eerw dssdfsselleds
Features
• usually presents in childhood, e.g. failure to thrivehe
• hypokalaemiahe

• normotensionhe

• weaknesshe

A 63-year-old man presents with lethargy. Examination is unremarkable apart from BP


192/112. Routine blood tests reveal:
he earaer aeraer asdsadas eerw dssdfsselleds
K+ 2.4 mmol/l
Bicarbonate 34 mmol/l

he earaer aeraer asdsadas eerw dssdfsselleds


What is the most likely diagnosis?ia

A.A Phaeochromocytomaia
B.A Cushing's syndromeia
C.A Diabetes mellitusia
D.A Bartter's syndromeia
x E.A Conn's syndromeia
Next question
Hypokalaemia associated with hypertension points towards a diagnosis of Conn's
syndrome. Cushing's syndrome causes both of these features but it would be unusual to
see this degree of hypokalaemia and an unremarkable clinical examination counts against
the diagnosis. Bartter's syndrome is associated with normotension
Primary hyperaldosteronism
sqweqwesf erwrewfsdfs adasd dhe
Primary hyperaldosteronism was previously thought to be most commonly caused by an
adrenal adenoma, termed Conn's syndrome. However, recent studies have shown that
bilateral idiopathic adrenal hyperplasia is the cause in up to 70% of cases. Differentiating
between the two is important as this determines treatment. Adrenal carcinoma is an
extremely rare cause of primary hyperaldosteronism
he earaer aeraer asdsadas eerw dssdfsselleds
Features
• hypertensionhe

• hypokalaemia (e.g. muscle weakness)he


• alkalosishe

he earaer aeraer asdsadas eerw dssdfsselleds


Investigations
• high serum aldosteronehe

• low serum reninhe


• high-resolution CT abdomenhe
he earaer aeraer asdsadas eerw dssdfsselleds
Management
• adrenal adenoma: surgeryhe

• bilateral adrenocortical hyperplasia: aldosterone antagonist e.g. spironolactonehe


Which one of the following is a recognised cause of hypokalaemia associated with
hypertensionia

x A.A Liddle's syndrome


B.A Bartter's syndromeia
C.A Gitelman syndromeia
D.A Ciclosporinia
E.A Renal tubular acidosisia
Next question
Hypokalaemia and hypertension
sqweqwesf erwrewfsdfs adasd dhe
For the MRCP it is useful to be able to classify the causes of hypokalaemia in to those
associated with hypertension, and those which are not
he earaer aeraer asdsadas eerw dssdfsselleds
Hypokalaemia with hypertension
• Cushing's syndromehe
• Conn's syndrome (primary hyperaldosteronism)he
• Liddle's syndromehe
• congenital adrenal hyperplasia (11-beta hydroxylase deficiency)he

he earaer aeraer asdsadas eerw dssdfsselleds


Carbenoxolone, an anti-ulcer drug, and liquorice excess can potentially cause
hypokalaemia associated with hypertension
he earaer aeraer asdsadas eerw dssdfsselleds
Hypokalaemia without hypertension
• diureticshe

• GI loss (e.g. diarrhoea, vomiting)he


• renal tubular acidosis (type 1 and 2*)he
• Bartter's syndromehe
• Gitelman syndromehe
he earaer aeraer asdsadas eerw dssdfsselleds
Urinary potassium may be helpful in diagnosis, if < 30mmol/day then GI loss or diuretic
use is likely to be the cause
he earaer aeraer asdsadas eerw dssdfsselleds
*type 4 renal tubular acidosis is associated with hyperkalaemia

Which one of the following features is least associated with primary


hyperparathyroidism?ia

A.A Depressionia
B.A Polydipsiaia
x C.A Sensory loss
D.A Peptic ulcerationia
E.A Hypertensionia
Next question
Primary hyperparathyroidism
sqweqwesf erwrewfsdfs adasd dhe
In the MRCP, primary hyperparathyroidism is stereotypically seen in elderly females with
an unquenchable thirst and an inappropriately normal or raised parathyroid hormone
level. It is most commonly due to a solitary adenoma
he earaer aeraer asdsadas eerw dssdfsselleds
Causes of primary hyperparathyroidism
• 80%: solitary adenomahe

• 15%: hyperplasiahe
• 4%: multiple adenomahe

• 1%: carcinomahe

he earaer aeraer asdsadas eerw dssdfsselleds


Features - 'bones, stones, abdominal groans and psychic moans'
• polydipsia, polyuriahe

• peptic ulceration/constipation/pancreatitis he
• bone pain/fracturehe

• renal stoneshe

• depressionhe

• hypertensionhe

• impaired glucose tolerancehe

he earaer aeraer asdsadas eerw dssdfsselleds


Associations
• hypertensionhe

• multiple endocrine neoplasia: MEN I and IIhe


he earaer aeraer asdsadas eerw dssdfsselleds
Investigations
• raised calcium, low phosphatehe
• PTH may be raised or normalhe
• technetium-MIBI subtraction scanhe
he earaer aeraer asdsadas eerw dssdfsselleds
Treatment
• total parathyroidectomyhe

A 45-year-old female is admitted to the Emergency Department with abdominal pain


associated with vomiting. She has a past medical history of hypothyroidism. On
examination she is pyrexial at 37.6ºC. Pulse is 110 bpm with a blood pressure of 100/64
mmHg. Blood results show the following:
he earaer aeraer asdsadas eerw dssdfsselleds
Na+ 131 mmol/l
K+ 4.9 mmol/l
Urea 8.1 mmol/l
Creatinine 110 µmol/l
Glucose 3.3 mmol/l

he earaer aeraer asdsadas eerw dssdfsselleds


What treatment should be given first?ia

A.A Ceftriaxone + benzylpenicillinia


B.A Glucagonia
C.A Propranololia
D.A Triiodothyronineia
x E.A Hydrocortisoneia
Next question
This is a typical history of Addison's. Steroids should be given as soon as possible
Addison's disease
sqweqwesf erwrewfsdfs adasd dhe
Autoimmune destruction of the adrenal glands is the commonest cause of
hypoadrenalism in the UK, accounting for 80% of cases
he earaer aeraer asdsadas eerw dssdfsselleds
Features
• lethargy, weakness, anorexia, N/V, weight losshe
• hyperpigmentation, vitiligo, loss of pubic hair in womenhe
• crisis: collapse, shock, pyrexiahe
he earaer aeraer asdsadas eerw dssdfsselleds
Other causes of hypoadrenalism
he earaer aeraer asdsadas eerw dssdfsselleds
Primary causes
• tuberculosishe

• metastases (e.g. bronchial carcinoma)he


• meningococcal septicaemia (Waterhouse-Friderichsen syndrome)he
• HIVhe

• antiphospholipid syndromehe
he earaer aeraer asdsadas eerw dssdfsselleds
Secondary causes
• pituitary disorders (e.g. tumours, irradiation, infiltration)he
he earaer aeraer asdsadas eerw dssdfsselleds
Exogenous glucocorticoid therapy
A 45-year-old man is referred to the acute medical unit. He had presented earlier in the
day to the GP complaining of ongoing fatigue and polydipsia. A BM taking in the surgery
was 22.3. On examination he is an obese man (BMI 36) with a pulse of 84 bpm and blood
pressure of 144/84 mmHg. Blood tests reveal the following:
he earaer aeraer asdsadas eerw dssdfsselleds
Na+ 140 mmol/l
K+ 3.9 mmol/l
Bicarbonate 22 mmol/l
Urea 5.2 mmol/l
Creatinine 101 µmol/l
Glucose 21.2 mmol/l

he earaer aeraer asdsadas eerw dssdfsselleds


What is the most appropriate initial management?ia
A.A Glicazideia
B.A Pioglitazoneia
C.A Weight lossia
x D.A Metforminia
E.A Commence insulin therapyia
Next question
Weight reduction alone would be insufficient in this patient with frank diabetes
Metformin
sqweqwesf erwrewfsdfs adasd dhe
Metformin is a biguanide used mainly in the treatment of type 2 diabetes mellitus. It has a
number of actions which improves glucose tolerance (see below). Unlike sulphonylureas
it does not cause hypoglycaemia and weight gain and is therefore first-line if the patient
is overweight. Another use of metformin is in polycystic ovarian syndrome where it has
been shown to increase ovulation and reduce hirsuitism
he earaer aeraer asdsadas eerw dssdfsselleds
Mechanism of action
• increases insulin sensitivityhe
• decreases hepatic gluconeogenesishe
• may also reduce gastrointestinal absorption of carbohydrateshe
he earaer aeraer asdsadas eerw dssdfsselleds
Adverse effects
• gastrointestinal upsets are common (nausea, anorexia, diarrhoea), intolerable in
20%he
• reduced vitamin B12 absorption - rarely a clinical problemhe
he earaer aeraer asdsadas eerw dssdfsselleds
Contraindications
• lactic acidosis with severe liver disease / renal failure / heart failurehe
• do not use during suspected episodes of tissue hypoxia (e.g. recent MI, sepsis)he
• alcohol abuse is a relative contraindicationhe
• pregnancyhe

• stop 2 days before GA, restart when renal function normalhe


• stop prior to IV contrast e.g. angiography, restart when renal function normalhe
A 56-year-old man is reviewed in the Cardiology outpatient clinic following a myocardial
infarction one year previously. During his admission he was found to be hypertensive and
diabetic. He complains that he has put on 10kg in weight in the past 6 months. Which of
his medications may be contributing to his weight gain?ia

A.A Metforminia
B.A Losartania
C.A Clopidogrelia
x D.A Glicazideia
E.A Simvastatinia
Next question
Sulfonylureas
sqweqwesf erwrewfsdfs adasd dhe
Sulfonylureas are oral hypoglycaemic drugs used in the management of type 2 diabetes
mellitus. They work by increasing pancreatic insulin secretion and hence are only
effective if functional B-cells are present.
he earaer aeraer asdsadas eerw dssdfsselleds
Common adverse effects
• hypoglycaemic episodes (more common with long acting preparations such as
chlorpropamidehe
• increased appetite and weight gainhe
he earaer aeraer asdsadas eerw dssdfsselleds
Rarer adverse effects
• syndrome of inappropriate ADH secretionhe
• bone marrow suppressionhe
• liver damage (cholestatic)he
• photosensitivityhe

• peripheral neuropathyhe

he earaer aeraer asdsadas eerw dssdfsselleds


Sulfonylureas should be avoided in breast feeding and pregnancy
Which one of the following statements regarding maturity-onset diabetes of the young
(MODY) is true?ia

x A.A There is usually a strong family history


B.A Body mass index is typically > 30ia
C.A Doesn’t respond to glimepirideia
D.A Autosomal recessive inheritanceia
E.A Frequent episodes of diabetic ketoacidosis are typicalia
Next question
MODY
sqweqwesf erwrewfsdfs adasd dhe
Maturity-onset diabetes of the young (MODY) is characterised by the development of
type 2 diabetes mellitus in patients < 25 years old. It is typically inherited as an
autosomal dominant condition. Over six different genetic mutations have so far been
identified as leading to MODY. Ketosis is not a feature at presentation
he earaer aeraer asdsadas eerw dssdfsselleds
MODY 3
• 60% of caseshe
• due to a defect in the HNF-1 alpha genehe
he earaer aeraer asdsadas eerw dssdfsselleds
MODY 2
• 20% of caseshe
• due to a defect in the glucokinase genehe
A 45-year-old man is investigated following referral to the endocrinology clinic with
polydipsia. Plasma glucose and calcium are normal. A water deprivation test is performed
with the following results:
he earaer aeraer asdsadas eerw dssdfsselleds
Starting plasma osm. 292 mOsmol/l
Final urine osm. 142 mOsmol/l
Urine osm. post-DDAVP 885 mOsmol/l

he earaer aeraer asdsadas eerw dssdfsselleds


What is the most likely diagnosis?ia

A.A Psychogenic polydipsiaia


B.A Nephrogenic diabetes insipidusia
C.A Primary hyperparathyroidismia
D.A Pseudohypoparathyroidismia
x E.A Cranial diabetes insipidus
Next question
A dramatic improvement is seen in the ability of the kidneys to concentrate urine
following the administration of DDAVP. This points towards a diagnosis of cranial
diabetes insipidus
Water deprivation test
sqweqwesf erwrewfsdfs adasd dhe
Method
• prevent patient drinking waterhe
• ask patient to empty bladderhe
• hourly urine and plasma osmolalitieshe
he earaer aeraer asdsadas eerw dssdfsselleds
Starting plasma Final urine Urine osm. post-
osm. osm. DDAVP

Normal Normal > 600 > 600

Polydipsia Low > 400 > 400

Cranial DI High < 300 > 600

Nephrogenic DI High < 300 < 300

A 46-year-old woman is referred to endocrine with a tender neck swelling. Blood results
are as follows:
he earaer aeraer asdsadas eerw dssdfsselleds
TSH <0.1 mU/l
T4 188 nmol/l

Hb 14.2 g/dl
Plt 377 * 109/l
WBC 6.4 * 109/l

ESR 65 mm/hr

he earaer aeraer asdsadas eerw dssdfsselleds


Technetium thyroid scan shows decreased uptake
he earaer aeraer asdsadas eerw dssdfsselleds
What is the most likely diagnosis?ia

A.A Sick thyroid syndromeia


B.A Acute bacterial thyroiditisia
C.A Hashimoto's thyroiditisia
x D.A Subacute thyroiditis
E.A Toxic multinodular goitreia
Next question
Subacute (De Quervain's) thyroiditis
sqweqwesf erwrewfsdfs adasd dhe
Subacute thyroiditis (also known as De Quervain's thyroiditis) is thought to occur
following viral infection and typically presents with hyperthyroidism
he earaer aeraer asdsadas eerw dssdfsselleds
Features
• hyperthyroidismhe

• painful goitrehe

• raised ESRhe

• globally reduced uptake on iodine-131 scanhe


he earaer aeraer asdsadas eerw dssdfsselleds
Management
• usually self-limiting - most patients do not require treatmenthe
• steroids may be used, particularly if hypothyroidism developshe
Each one of the following is a cause of cranial diabetes insipidus, except:ia

A.A Pituitary surgeryia


x B.A Lithiumia
C.A Histiocytosis Xia
D.A Craniopharyngiomaia
E.A Post head-injuryia
Next question
Lithium causes a nephrogenic diabetes insipidus
Diabetes insipidus
sqweqwesf erwrewfsdfs adasd dhe
Causes of cranial DI
• idiopathiche

• post head injuryhe


• pituitary surgeryhe
• craniopharyngiomashe

• histiocytosis Xhe
he earaer aeraer asdsadas eerw dssdfsselleds
DIDMOAD is the association of cranial Diabetes Insipidus, Diabetes Mellitus, Optic
Atrophy and Deafness (also known as Wolfram's syndrome)
he earaer aeraer asdsadas eerw dssdfsselleds
Causes of nephrogenic DI
• genetic (primary)he

• electrolytes: hypercalcaemia, hypokalaemiahe


• drugs: demeclocycline, lithiumhe
• tubulo-interstitial disease: obstruction, sickle-cell, pyelonephritishe
he earaer aeraer asdsadas eerw dssdfsselleds
Investigation
• high plasma osmolarity, low urine osmolarityhe
• water deprivation testhe
Which one of the following neonatal complications is least commonly seen in diabetic
pregnancies?ia

A.A Stillbirthia
B.A Respiratory distress syndromeia
x C.A Microsomiaia
D.A Hypoglycaemiaia
E.A Sacral agenesisia
Next question
Diabetes predisposes to macrosomia rather than microsomia
Pregnancy: diabetes - complications
sqweqwesf erwrewfsdfs adasd dhe
Maternal complications
• polyhydramnios - 25%, possibly due to fetal polyuriahe
• preterm labour - 15%, associated with polyhydramnioshe
he earaer aeraer asdsadas eerw dssdfsselleds
Neonatal complications
• macrosomiahe

• hypoglycaemiahe

• respiratory distress syndrome: surfactant production is delayedhe


• polycythaemia: therefore more neonatal jaundicehe

• malformation ratesincrease 3-4 fold e.g. sacral agenesis, CNS and CVS
malformations (HOCM)he
• stillbirthhe

• hypomagnesaemiahe

• hypocalcaemiahe

• shoulder dystociahe
A 29 year female presents abdominal striae, hirsuitism and myopathy. A diagnosis of
Cushing's syndrome is suspected. Which one of the following is the most appropriate test
to confirm the diagnosis?ia

A.A High-dose dexamethasone suppression testia


B.A Plasma ACTHia
C.A Short ACTH testia
x D.A 24 hour urinary free cortisol
E.A Petrosal sinus sampling of ACTHia
Next question
The overnight dexamethasone suppression test is the best test to diagnosis Cushing's
syndrome
There is some debate as to whether a 24 hour urinary free cortisol or an overnight
dexamethasone suppression test should be used to screen patients for Cushing's. The
overnight dexamethasone suppression test has however been shown to be more sensitive
and is now much more commonly used in clinical practice. As this is not offered then 24
hour urinary free cortisol is the next best answer
he earaer aeraer asdsadas eerw dssdfsselleds
The high-dose dexamethasone suppression test is used to help differentiate the cause of
Cushing's syndrome
Cushing's syndrome: investigations
sqweqwesf erwrewfsdfs adasd dhe
Investigations are divided into confirming Cushing's syndrome and then localising the
lesion. A hypokalaemic metabolic alkalosis may be seen, along with impaired glucose
tolerance. Ectopic ACTH secretion (e.g. secondary to small cell lung cancer) is
characteristically associated with very low potassium levels. An insulin stress test is used
to differentiate between true Cushing's and pseudo-Cushing's
he earaer aeraer asdsadas eerw dssdfsselleds
Tests to confirm Cushing's syndrome
he earaer aeraer asdsadas eerw dssdfsselleds
The two most commonly used tests are:
• overnight dexamethasone suppression test (most sensitive)he
• 24 hr urinary free cortisolhe
he earaer aeraer asdsadas eerw dssdfsselleds
Localisation tests
he earaer aeraer asdsadas eerw dssdfsselleds
The first-line localisation is 9am and midnight plasma ACTH (and cortisol) levels. If
ACTH is suppressed then a non-ACTH dependent cause is likely such as an adrenal
adenoma
he earaer aeraer asdsadas eerw dssdfsselleds
High-dose dexamethasone suppression test
• if pituitary source then cortisol suppressedhe
• if ectopic/adrenal then no change in cortisolhe
he earaer aeraer asdsadas eerw dssdfsselleds
CRH stimulation
• if pituitary source then cortisol riseshe
• if ectopic/adrenal then no change in cortisolhe
he earaer aeraer asdsadas eerw dssdfsselleds
Petrosal sinus sampling of ACTH may be needed to differentiate between pituitary and
ectopic ACTH secretion

A 46-year-old man presents to his GP concerned about reduced libido and excessive
thirst. On questioning he also complains of pain in his right hip. Which one of the
following investigations is most likely to reveal the diagnosis?ia

x A.A Ferritinia
B.A Testosteroneia
C.A Cortisolia
D.A Blood glucoseia
E.A Prolactinia
Next question
The above patient has symptoms consistent with haemochromatosis. The excessive thirst
is secondary to untreated diabetes mellitus
Haemochromatosis: features
sqweqwesf erwrewfsdfs adasd dhe
Haemochromatosis is an autosomal recessive disorder of iron absorption and metabolism
resulting in iron accumulation. It is caused by inheritance of mutations in the HFE gene
on both copies of chromosome 6*. It is often asymptomatic in early disease and initial
symptoms often non-specific e.g. lethargy and arthralgia
he earaer aeraer asdsadas eerw dssdfsselleds
Presenting features
• 'bronze' skin pigmentationhe
• diabetes mellitushe
• liver: stigmata of chronic liver disease, hepatomegaly, cirrhosis, hepatocellular
deposition)he
• cardiac failure (2nd to dilated cardiomyopathy)he
• hypogonadism (2nd to cirrhosis and pituitary dysfunction - hypogonadotrophic
hypogonadism)he
he earaer aeraer asdsadas eerw dssdfsselleds
Questions have previously been asked regarding which features are reversible with
treatment:
he earaer aeraer asdsadas eerw dssdfsselleds
Reversible complications Irreversible complications
• Cardiomyo • Liver cirrhosishe
• pathyhe • Diabetes mellitushe
• Skin pigmentationhe • Hypogonadotrophic hypogonadismhe

• Arthropathy

He
he earaer aeraer asdsadas eerw dssdfsselleds
*there are rare cases of families with classic features of genetic haemochromatosis but no
mutation in the HFE gene

The following cause a rise in triglyceride levels, as opposed to cholesterol levels,


except:ia

A.A Diabetes mellitusia


B.A Bendrofluazideia
x C.A Nephrotic syndrome
D.A Alcoholia
E.A Obesityia
Next question
Hyperlipidaemia: secondary causes
sqweqwesf erwrewfsdfs adasd dhe
Causes of predominately hypertriglyceridaemia
• diabetes mellitus (types 1 and 2)he
• obesityhe

• alcoholhe

• chronic renal failurehe

• drugs: thiazides, non-selective beta-blockers, unopposed oestrogenhe


• liver diseasehe
he earaer aeraer asdsadas eerw dssdfsselleds
Causes of predominately hypercholesterolaemia
• nephrotic syndromehe

• cholestasishe

• hypothyroidismhe

In patients with suspected insulinoma, which one of the following is considered the best
investigation?ia

A.A Oral glucose tolerance testia


B.A Insulin tolerance testia
C.A Early morning C-peptide levelsia
D.A Glucagon stimulation testia
x E.A Supervised fasting
Next question
Insulinoma is diagnosed with supervised prolonged fasting
CT of the pancreas is also useful in demonstrating a lesion
Insulinoma
sqweqwesf erwrewfsdfs adasd dhe
Basics
• tumour of Islets of Langerhanshe
• most common pancreatic endocrine tumourhe

• 10% malignant, 10% multiplehe


• of patients with multiple tumours, 50% have MEN-1he
he earaer aeraer asdsadas eerw dssdfsselleds
Features
• of hypoglycaemia either early in morning or just before mealhe
• high insulin, raised proinsulin:insulin ratiohe
• high C-peptidehe

he earaer aeraer asdsadas eerw dssdfsselleds


Diagnosis
• supervised, prolonged fasting (up to 72 hours)he
• CT pancreashe

A 65-year-old man with a history of dyspepsia is found to have a gastric MALT


lymphoma on biopsy. What treatment should be offered?ia

A.A Gastrectomyia
B.A Laser ablationia
C.A Noneia
D.A CHOP chemotherapyia
x E.A H. pylori eradication
Next question
Gastric MALT lymphoma
sqweqwesf erwrewfsdfs adasd dhe
Overview
• associated with H. pylori infection in 95% of caseshe
• good prognosishe
• if low grade then 80% respond to H. Pylori eradicationhe
he earaer aeraer asdsadas eerw dssdfsselleds
Features
• paraproteinaemia may be presenthe
What is the mechanism of action of thiazolidinediones?ia

A.A PPAR-gamma receptor antagonistia


B.A PPAR-alpha receptor antagonistia
C.A PPAR-alpha receptor agonistia
x D.A PPAR-gamma receptor agonist
E.A Increases endogenous insulin secretionia
Next question
Glitazones are agonists of PPAR-gamma receptors
Thiazolidinediones
sqweqwesf erwrewfsdfs adasd dhe
Thiazolidinediones are a new class of agents used in the treatment of type 2 diabetes
mellitus. They are agonists to the PPAR-gamma receptor and reduce peripheral insulin
resistance
he earaer aeraer asdsadas eerw dssdfsselleds
The PPAR-gamma receptor is an intracellular nuclear receptor. Its natural ligands are free
fatty acids and it is thought to control adipocyte differentiation and function
he earaer aeraer asdsadas eerw dssdfsselleds
Adverse effects
• weight gainhe

• liver impairment: monitor LFTshe


• fluid retention - therefore contraindicated in heart failurehe
Which one of the following types of thyroid cancer is seen in multiple endocrine
neoplasia (MEN) type 2?ia

A.A Anaplasticia
B.A Papillaryia
x C.A Medullaryia
D.A Follicularia
E.A Lymphomaia
Next question
Thyroid cancer
sqweqwesf erwrewfsdfs adasd dhe
Features of hyperthyroidism or hypothyroidism are not commonly seen in patients with
thyroid malignancies as they rarely secrete thyroid hormones
he earaer aeraer asdsadas eerw dssdfsselleds
Type Percentage
Papillary 70% Often young females - excellent prognosis
Follicular 20%
Medullary 5% Cancer of parafollicular cells, secrete calcitonin, part
of MEN-2
Anaplastic 1% Not responsive to treatment, can cause pressure
symptoms
Lymphoma Rare Associated with Hashimoto's

he earaer aeraer asdsadas eerw dssdfsselleds


Management of papillary and follicular cancer
• total thyroidectomyhe

• followed by radioiodine (I-131) to kill residual cellshe


• yearly thyroglobulin levels to detect early recurrent diseasehe
A 45-year-old female presents to her GP with a two month history of lethargy. Blood tests
reveal the following:
he earaer aeraer asdsadas eerw dssdfsselleds
Na+ 129 mmol/l
K+ 5.1 mmol/l
Urea 5.3 mmol/l
Creatinine 99 µmol/l

T4 66 mmol/l

he earaer aeraer asdsadas eerw dssdfsselleds


Which one of the following investigations is most likely to reveal the diagnosis?ia

A.A Serum glucoseia


B.A TSHia
C.A Free T4ia
D.A Overnight dexamethasone suppression testia
x E.A Short synacthen testia
Next question
The short synacthen test is the best test to diagnose Addison's disease
Hyponatraemia and hyperkalaemia in a patient with lethargy is highly suggestive of
Addison's disease. The thyroxine level is slightly low and she may indeed have co-
existing hypothyroidism but this would not explain the hyperkalaemia
Addison's disease: investigations
sqweqwesf erwrewfsdfs adasd dhe
In a patient with suspect Addison's disease the definite investigation is a short ACTH test.
Plasma cortisol is measure before and 30 minutes after giving Synacthen 250ug IM.
Adrenal autoantibodies such as anti-21-hydroxylase may also be demonstrated
he earaer aeraer asdsadas eerw dssdfsselleds
Associated electrolyte abnormalities
• hyperkalaemiahe

• hyponatraemiahe
• hypoglycaemiahe

• metabolic acidosishe
Which one of the following is not associated with primary hyperparathyroidism?ia

x A.A Hypotensionia
B.A Multiple endocrine neoplasia type 1ia
C.A Multiple endocrine neoplasia type 2aia
D.A Depressionia
E.A Pancreatitisia
Next question
Primary hyperparathyroidism is associated with hypertension
Primary hyperparathyroidism
sqweqwesf erwrewfsdfs adasd dhe
In the MRCP, primary hyperparathyroidism is stereotypically seen in elderly females with
an unquenchable thirst and an inappropriately normal or raised parathyroid hormone
level. It is most commonly due to a solitary adenoma
he earaer aeraer asdsadas eerw dssdfsselleds
Causes of primary hyperparathyroidism
• 80%: solitary adenomahe
• 15%: hyperplasiahe

• 4%: multiple adenomahe


• 1%: carcinomahe

he earaer aeraer asdsadas eerw dssdfsselleds


Features - 'bones, stones, abdominal groans and psychic moans'
• polydipsia, polyuriahe

• peptic ulceration/constipation/pancreatitis he
• bone pain/fracturehe

• renal stoneshe
• depressionhe

• hypertensionhe
• impaired glucose tolerancehe

he earaer aeraer asdsadas eerw dssdfsselleds


Associations
• hypertensionhe

• multiple endocrine neoplasia: MEN I and IIhe


he earaer aeraer asdsadas eerw dssdfsselleds
Investigations
• raised calcium, low phosphatehe
• PTH may be raised or normalhe
• technetium-MIBI subtraction scanhe
he earaer aeraer asdsadas eerw dssdfsselleds
Treatment
• total parathyroidectomyhe

A 36-year-old female with a BMI of 34 presents to her GP after managing to lose 3 kg in


the past month. She asks about the possibility of starting a drug to help her lose weight.
What is the primary mode of action of orilistat?ia

A.A Leptin antagonistia


x B.A Pancreatic lipase inhibitor
C.A Blocks intestinal absorption of lipidsia
D.A HMG-CoA reductase inhibitoria
E.A Centrally-acting appetite suppressantia
Next question
The primary mode of action of orilistat is to inhibit pancreatic lipases, which in turn will
decrease the absorption of lipids from the intestine
Obesity: therapeutic options
sqweqwesf erwrewfsdfs adasd dhe
The management of obesity consists of step-wise approach:
• conservative: diet, exercisehe
• medicalhe
• surgicalhe

he earaer aeraer asdsadas eerw dssdfsselleds


Orlistat
• a pancreatic lipase inhibitor used in the management of obesity. NICE has defined
strict criteria about who should get the drughe
• adverse effects include faecal urgency/incontinence, flatulencehe
he earaer aeraer asdsadas eerw dssdfsselleds
NICE criteria for use of orlistat
• prior weight loss if >= 2.5 kg in the month prior to the prescriptionhe
• BMI > 28 plus comorbidities (e.g. diabetes, hypertension) orhe
• BMI > 30 he
• continued weight loss e.g. 5% at 3 months, 10% at 6 monthshe
• normally used for < 1 year, maximum duration of treatment = 2 yearshe
he earaer aeraer asdsadas eerw dssdfsselleds
Sibutramine
• centrallyacting appetite suppressant (inhibitor of serotonin and noradrenaline
uptake at hypothalamic sites that regular food intake)he
• adverse effects include: constipation, headache, dry mouth, insomnia and
anorexiahe
• contraindicated in hypertension, IHD, stroke, arrhythmiashe
he earaer aeraer asdsadas eerw dssdfsselleds
Rimonabant
• a specific CB1 cannabinoid receptor antagonisthe
• helps reduce the craving for foodhe
• adverse effects include depression, anxiety and insomniahe
A 28-year-old woman with polycystic ovarian syndrome consults you as she is having
problems becoming pregnant. Which one of the following drugs may increase fertility in
patients with polycystic ovarian syndrome?ia

A.A Ciclosporinia
B.A Bromocriptineia
C.A Glicazideia
x D.A Metforminia
E.A Acarboseia
Next question
Whilst diet and exercise and important in increasing the fertility of patients with
polycystic ovarian syndrome (PCOS) this is often not enough alone. Metformin has been
shown to induce ovulation in up to 50% of patients with PCOS
Polycystic ovarian syndrome
sqweqwesf erwrewfsdfs adasd dhe
Features
• hirsuitism, acnehe
• amenorrhoea, infertilityhe

• obesityhe
• acanthosis nigricanshe
• hypertensionhe
he earaer aeraer asdsadas eerw dssdfsselleds
Investigations
• transvaginal ultrasound of the pelvishe
• impaired glucose tolerancehe

• raised LH/FSH ratiohe


• raised prolactinhe

• mildly raised testosterone - however, if markedly raised consider other causeshe


• positive (withdrawal bleed) response to Provera challengehe
he earaer aeraer asdsadas eerw dssdfsselleds
Management
• metformin: improves both insulin sensitivity and increases chances of conceptionhe
A 45-year-old man is presents with bitemporal hemianopia and spade-like hands. What is
the definite test to confirm the diagnosis?ia

A.A Early morning growth hormoneia


B.A Insulin tolerance testia
x C.A Oral glucose tolerance with growth hormone measurements
D.A Random insulin-like growth factor 1 (IGF-!)ia
E.A Short ACTH testia
Next question
The diagnostic test for acromegaly is an oral glucose tolerance with growth hormone
measurements
Acromegaly: investigations
sqweqwesf erwrewfsdfs adasd dhe
Growth hormone (GH) levels vary during the day and are therefore not diagnostic. The
definitive test is the oral glucose tolerance (OGTT) with serial GH measurements. The
serum IGF-1 may also be used as a screening test and is sometimes used to monitor
disease
he earaer aeraer asdsadas eerw dssdfsselleds
Oral glucose tolerance test
• in normal patients GH is suppressed to < 2 mu/L with hyperglycaemiahe
• in acromegaly there is no suppression of GHhe
• may also demonstrate impaired glucose tolerance which is associated with
acromegalyhe
he earaer aeraer asdsadas eerw dssdfsselleds
A pituitary MRI may demonstrate a pituitary tumour
A 35-year-old female is referred to the endocrine clinic due to weight loss and
palpitations. The following results are obtained:
he earaer aeraer asdsadas eerw dssdfsselleds
TSH < 0.05 mu/l
T4 178 mmol/l

he earaer aeraer asdsadas eerw dssdfsselleds


Which one of the following features would most suggest a diagnosis of Grave's disease?
ia

A.A Atrial fibrillationia


B.A Lid lagia
C.A Family history of radioiodine treatmentia
x D.A Pretibial myxoedema
E.A Multinodular goitreia
Next question
Pretibial myxoedema is not seen in other causes of thyrotoxicosis and points towards a
diagnosis of Grave's
Grave's disease
sqweqwesf erwrewfsdfs adasd dhe
Features seen in Grave's but not in other causes of thyrotoxicosis
• eye signs: exophthalmos, ophthalmoplegiahe
• pretibial myxoedemahe

• thyroid acropachyhe
he earaer aeraer asdsadas eerw dssdfsselleds
Autoantibodies
• anti-TSH receptor stimulating antibodies (90%)he
• anti-thyroid peroxidase (microsomal) antibodies (50%)he
Which of the following is least recognised as a potential complication of acromegaly?ia

A.A Colorectal canceria


B.A Hypertensionia
C.A Cardiomyopathyia
D.A Diabetes mellitusia
x E.A Pulmonary hypertension
Next question
Secondary causes of pulmonary hypertension include COPD, congenital heart disease
(Eisenmenger's syndrome), recurrent pulmonary embolism, HIV and sarcoidosis
Acromegaly: features
sqweqwesf erwrewfsdfs adasd dhe
In acromegaly there is excess growth hormone secondary to a pituitary adenoma in over
95% of cases. A minority of cases are caused by ectopic GHRH or GH production by
tumours e.g. pancreatic
he earaer aeraer asdsadas eerw dssdfsselleds
Features
• course, oily skin , large tongue, prognathism, interdental spaceshe
• spade-like hands, increase in shoe sizehe
• features of pituitary tumour: hypopituitarism, headaches, bitemporal hemianopiahe
• raised prolactin in 1/3 of cases --> galactorrhoeahe
• 6% of patients have MEN-1he
he earaer aeraer asdsadas eerw dssdfsselleds
Complications
• hypertensionhe
• diabetes (>10%)he
• cardiomyopathyhe
• colorectal cancerhe

Each one of the following is a cause of nephrogenic diabetes insipidus, except:ia

A.A Hypercalcaemiaia
B.A Demeclocyclineia
x C.A Histiocytosis Xia
D.A Lithiumia
E.A Hypokalaemiaia
Next question
Diabetes insipidus
sqweqwesf erwrewfsdfs adasd dhe
Causes of cranial DI
• idiopathiche

• post head injuryhe


• pituitary surgeryhe
• craniopharyngiomashe
• histiocytosis Xhe
he earaer aeraer asdsadas eerw dssdfsselleds
DIDMOAD is the association of cranial Diabetes Insipidus, Diabetes Mellitus, Optic
Atrophy and Deafness (also known as Wolfram's syndrome)
he earaer aeraer asdsadas eerw dssdfsselleds
Causes of nephrogenic DI
• genetic (primary)he
• electrolytes: hypercalcaemia, hypokalaemiahe
• drugs: demeclocycline, lithiumhe
• tubulo-interstitial disease: obstruction, sickle-cell, pyelonephritishe
he earaer aeraer asdsadas eerw dssdfsselleds
Investigation
• high plasma osmolarity, low urine osmolarityhe
• water deprivation testhe
Each one of the following is associated with pseudohypoparathyroidism, except:ia

A.A Low calcium levelsia


x B.A Low PTH levels
C.A Shortened 4th and 5th metacarpalsia
D.A Low IQia
E.A Short statureia
Next question
Hypoparathyroidism
sqweqwesf erwrewfsdfs adasd dhe
Primary hypoparathyroidism
• decrease PTH secretionhe
• e.g. secondary to thyroid surgeryhe
• low calcium, high phosphatehe
• treat with alfacalcidolhe
he earaer aeraer asdsadas eerw dssdfsselleds
Pseudohypoparathyroidism
• target cells being insensitive to PTHhe
• due to abnormality in a G proteinhe
• associated with low IQ, short stature, shortened 4th and 5th metacarpalshe
• low calcium, high phosphate, high PTHhe
• diagnosis is made by measuring urinary cAMP and phosphate levels following an
infusion of PTH. In hypoparathyroidism this will cause an increase in both cAMP
and phosphate levels. In pseudohypoparathyroidism type I neither cAMP nor
phosphate levels are increased whilst in pseudohypoparathyroidism type II only
cAMP rises.he
he earaer aeraer asdsadas eerw dssdfsselleds
Pseudopseudohypoparathyroidism
• similar phenotype to pseudohypoparathyroidism but normal biochemistryhe
An insulin stress test is most useful in the investigation of:ia

A.A Glucagonomaia
B.A Insulinomaia
C.A Addison's diseaseia
x D.A Hypopituitarism
E.A Diabetes mellitusia
Next question
Insulin stress tests are also occasionally used to differentiate Cushing's from pseudo-
Cushing's
Insulin stress test
sqweqwesf erwrewfsdfs adasd dhe
Basics
• used in investigation of hypopituitarismhe
• IV insulin given, GH and cortisol levels measuredhe
• with normal pituitary function GH and cortisol should risehe
he earaer aeraer asdsadas eerw dssdfsselleds
Contraindications
• epilepsyhe
• ischaemic heart diseasehe

• adrenal insufficiencyhe
A 56-year-old female is admitted to ITU with a severe pneumonia. Thyroid function tests
are most likely to show:ia

A.A TSH normal; thyroxine high; T3 highia


x B.A TSH low; thyroxine low; T3 low
C.A TSH high; thyroxine low; T3 lowia
D.A TSH low; thyroxine high; T3 highia
E.A TSH high; thyroxine normal; T3 highia
Next question
Sick euthyroid syndrome
sqweqwesf erwrewfsdfs adasd dhe
In sick euthyroid syndrome it is often said that everything (TSH, thyroxine and T3) is
low. The thyroxine level may however be normal
he earaer aeraer asdsadas eerw dssdfsselleds
Changes are reversible upon recovery from the systemic illness
Liddle's syndrome is associated with each one of the following, except:ia

A.A Alkalosisia
B.A Response to treatment with amilorideia
C.A Autosomal dominant inheritanceia
x D.A Normotensionia
E.A Hypokalaemiaia
Next question
Liddle's syndrome
sqweqwesf erwrewfsdfs adasd dhe
Overview
• autosomal dominant conditionhe
• causes hypertension and hypokalaemic alkalosishe
• thought tobe caused by a disorder sodium channels in the distal tubules leading to
increased reabsorption of sodium he
• treatment is with amiloridehe
Which of the following results establishes a diagnosis of diabetes mellitus?ia

A.A Asymptomatic patient with fasting glucose 7.9 mmol/L on one occasionia
B.A Symptomatic patient with fasting glucose 6.8 mmol/L on two occasionsia
C.A Glycosuria +++ia
D.A Asymptomatic patient with random glucose 22.0 mmol/L on one
occasionia
x E.A Symptomatic patient with random glucose 12.0 mmol/L on one occasion
Next question
Diabetes diagnosis: fasting > 7.0, random > 11.1 - if asymptomatic need two readings
Diabetes mellitus: diagnosis
sqweqwesf erwrewfsdfs adasd dhe
Based on 1997 American Diabetes Association (ADA) guidelines
he earaer aeraer asdsadas eerw dssdfsselleds
If symptomatic
• fasting glucose > 7.0 mmol/lhe
• random glucose > 11.1 mmol/l (or after 75g oral glucose tolerance test)he
he earaer aeraer asdsadas eerw dssdfsselleds
If asymptomatic above criteria apply but must be repeated on two occasions
he earaer aeraer asdsadas eerw dssdfsselleds
Impaired fasting glucose and impaired glucose tolerance
he earaer aeraer asdsadas eerw dssdfsselleds
A fasting glucose >= 6.1 but < 7.0 mmol/l implies impaired fasting glucose (IFG)
he earaer aeraer asdsadas eerw dssdfsselleds
Impaired glucose tolerance (IGT) is defined as fasting plasma glucose < 7.0 mmol/l and
OGTT 2-hour value >= 7.8 mmol/l but < 11.1 mmol/l
he earaer aeraer asdsadas eerw dssdfsselleds
Diabetes UK suggests:
• 'People with IFG should then be offered an oral glucose tolerance test to rule out a
diagnosis of diabetes. A result below 11.1 mmol/l but above 7.8 mmol/l indicates
that the person doesn’t have diabetes but does have IGT.'he
A 27-year-old man with multiple pigmented freckles on his lips and face is investigated
for iron-deficiency anaemia. A diagnosis of Peutz-Jeghers syndrome is suspected. What
is the mode of inheritance?ia

A.A Autosomal recessiveia


B.A Mitochondrial inheritanceia
C.A X-linked dominantia
x D.A Autosomal dominant
E.A X-linked recessiveia
Next question
Peutz-Jeghers syndrome
sqweqwesf erwrewfsdfs adasd dhe
Peutz-Jeghers syndrome is an inherited condition characterised by numerous
hamartomatous polyps in GI tract (mainly small bowel) associated with pigmented
freckles on the lips, face, palms and soles. The polyps undergo malignant change in less
than 3% of patients
he earaer aeraer asdsadas eerw dssdfsselleds
Genetics
• autosomal dominanthe

• responsible gene encodes serine threonine kinase LKB1 or STK11he


he earaer aeraer asdsadas eerw dssdfsselleds
Features
• hamartomatous polyps in GI tracthe
• pigmented lesions on lips, oral mucosa, face, palms and soleshe
• intestinal obstruction e.g. intussusceptionhe
• GI bleedinghe
he earaer aeraer asdsadas eerw dssdfsselleds
Management
• conservative unless complications develophe
Which one of the following may be associated with galactorrhoea?ia

x A.A Primary hypothyroidism


B.A Addison's diseaseia
C.A Cushing's syndromeia
D.A Grave's diseaseia
E.A Bromocriptineia
Next question
Prolactin and galactorrhoea
sqweqwesf erwrewfsdfs adasd dhe
Prolactin is secreted by the anterior pituitary gland with release being controlled by a
wide variety of physiological factors. Dopamine acts as the primary prolactin releasing
inhibitory factor and hence dopamine agonists such as bromocriptine may be used to
control galactorrhoea. It is important to differentiate the causes of galactorrhoea (due to
the actions of prolactin on breast tissue) from those of gynaecomastia
he earaer aeraer asdsadas eerw dssdfsselleds
Features of excess prolactin
• men: impotence, loss of libido, galactorrhoeahe
• women: amenorrhoea, galactorrhoeahe
he earaer aeraer asdsadas eerw dssdfsselleds
Causes of raised prolactin
• pregnancyhe
• oestrogenshe

• stresshe
• exercisehe

• sleephe
• acromegaly: 1/3 of patientshe
• polycystic ovarian syndromehe
• primary hypothyroidism (due to thyrotrophin releasing hormone (TRH) stimulating
prolactin release)he
he earaer aeraer asdsadas eerw dssdfsselleds
Drug causes of raised prolactin
• metoclopramide, domperidonehe
• phenothiazineshe
• haloperidolhe

• very rare: SSRIs, opioidshe


What is the most common cause of primary hyperparathyroidism?ia

A.A Idiopathicia
B.A Carcinomaia
C.A Hyperplasiaia
x D.A Solitary adenoma
E.A Multiple adenomaia
Next question
Primary hyperparathyroidism
sqweqwesf erwrewfsdfs adasd dhe
In the MRCP, primary hyperparathyroidism is stereotypically seen in elderly females with
an unquenchable thirst and an inappropriately normal or raised parathyroid hormone
level. It is most commonly due to a solitary adenoma
he earaer aeraer asdsadas eerw dssdfsselleds
Causes of primary hyperparathyroidism
• 80%: solitary adenomahe

• 15%: hyperplasiahe
• 4%: multiple adenomahe

• 1%: carcinomahe
he earaer aeraer asdsadas eerw dssdfsselleds
Features - 'bones, stones, abdominal groans and psychic moans'
• polydipsia, polyuriahe
• peptic ulceration/constipation/pancreatitis he
• bone pain/fracturehe
• renal stoneshe

• depressionhe
• hypertensionhe

• impaired glucose tolerancehe


he earaer aeraer asdsadas eerw dssdfsselleds
Associations
• hypertensionhe

• multiple endocrine neoplasia: MEN I and IIhe


he earaer aeraer asdsadas eerw dssdfsselleds
Investigations
• raised calcium, low phosphatehe
• PTH may be raised or normalhe
• technetium-MIBI subtraction scanhe
he earaer aeraer asdsadas eerw dssdfsselleds
Treatment
• total parathyroidectomyhe

A 44-year-old man with a BMI of 37 presents to his GP. Despite diet and exercise he has
failed to lose a significant amount of weight. His GP considers starting sibutramine. What
is the mechanism of action of sibutramine?ia

x A.A Serotonin and noradrenaline uptake inhibitor


B.A Leptin antagonistia
C.A CB1 cannabinoid receptor antagonistia
D.A Blocks intestinal absorption of lipidsia
E.A Pancreatic lipase inhibitoria
Next question
Obesity: therapeutic options
sqweqwesf erwrewfsdfs adasd dhe
The management of obesity consists of step-wise approach:
• conservative: diet, exercisehe
• medicalhe

• surgicalhe
he earaer aeraer asdsadas eerw dssdfsselleds
Orlistat
• a pancreatic lipase inhibitor used in the management of obesity. NICE has defined
strict criteria about who should get the drughe
• adverse effects include faecal urgency/incontinence, flatulencehe
he earaer aeraer asdsadas eerw dssdfsselleds
NICE criteria for use of orlistat
• prior weight loss if >= 2.5 kg in the month prior to the prescriptionhe
• BMI > 28 plus comorbidities (e.g. diabetes, hypertension) orhe
• BMI > 30 he
• continued weight loss e.g. 5% at 3 months, 10% at 6 monthshe
• normally used for < 1 year, maximum duration of treatment = 2 yearshe
he earaer aeraer asdsadas eerw dssdfsselleds
Sibutramine
• centrallyacting appetite suppressant (inhibitor of serotonin and noradrenaline
uptake at hypothalamic sites that regular food intake)he
• adverse effects include: constipation, headache, dry mouth, insomnia and
anorexiahe
• contraindicated in hypertension, IHD, stroke, arrhythmiashe
he earaer aeraer asdsadas eerw dssdfsselleds
Rimonabant
• a specific CB1 cannabinoid receptor antagonisthe
• helps reduce the craving for foodhe
• adverse effects include depression, anxiety and insomniahe
A 56-year-old lady with a BMI of 27 is reviewed in the diabetic clinic due to poor
glycaemic control. She is currently being treated with glicazide 160mg bd. Her latest
bloods show:
he earaer aeraer asdsadas eerw dssdfsselleds
Na+ 139 mmol/l
K+ 4.1 mmol/l
Urea 8.4 mmol/l
Creatinine 160 µmol/l
ALT 25 iu/l
yGT 33 iu/l
HbA1c 9.4%

he earaer aeraer asdsadas eerw dssdfsselleds


Which one of the following medications should be added next?ia

A.A Guar gumia


x B.A Rosiglitazoneia
C.A Metforminia
D.A Acarboseia
E.A Repaglinideia
Next question
Given her obesity metformin would be a natural choice to add. The creatinine however is
elevated so rosiglitazone is the next best option
he earaer aeraer asdsadas eerw dssdfsselleds
One possible drawback of using rosiglitazone is that it may lead to weight gain,
especially given her BMI is already 27.
Thiazolidinediones
sqweqwesf erwrewfsdfs adasd dhe
Thiazolidinediones are a new class of agents used in the treatment of type 2 diabetes
mellitus. They are agonists to the PPAR-gamma receptor and reduce peripheral insulin
resistance
he earaer aeraer asdsadas eerw dssdfsselleds
The PPAR-gamma receptor is an intracellular nuclear receptor. Its natural ligands are free
fatty acids and it is thought to control adipocyte differentiation and function
he earaer aeraer asdsadas eerw dssdfsselleds
Adverse effects
• weight gainhe

• liver impairment: monitor LFTshe


• fluid retention - therefore contraindicated in heart failurehe
What is the most common hormone secreted from pituitary tumours?ia

A.A TSHia
B.A LHia
C.A GHia
x D.A Prolactinia
E.A ACTHia
Pituitary tumours
sqweqwesf erwrewfsdfs adasd dhe
Hormones secreted
• PRL - 35%he
• no obvious hormone, 'non-functioning', 'chromophobe' - 30%he
• GH - 20%he
• PRL and GH - 7%he
• ACTH - 7%he
• others: TSH, LH, FSH - 1%he
Which one of the following is least associated with gynaecomastia?ia

A.A Klinefelter's syndromeia


B.A Seminomaia
C.A Liver diseaseia
D.A Pubertyia
x E.A Hypothyroidismia
Next question
Gynaecomastia is seen in up to a third of men with thyrotoxicosis, but is not a feature of
hypothyroidism
Gynaecomastia
sqweqwesf erwrewfsdfs adasd dhe
Gynaecomastia describes an abnormal amount of breast tissue in males and is usually
caused by an increased oestrogen:androgen ratio. It is important to differentiate the
causes of galactorrhoea (due to the actions of prolactin on breast tissue) from those of
gynaecomastia
he earaer aeraer asdsadas eerw dssdfsselleds
Causes of gynaecomastia
• physiological: normal in pubertyhe
• syndromes with androgen deficiency: Kallman's, Klinefelter'she
• testicular failure: e.g. mumpshe
• liver diseasehe
• testicular cancer e.g. seminoma secreting hCGhe
• ectopic tumour secretionhe
• hyperthyroidismhe
• haemodialysishe

• drugs: see belowhe


he earaer aeraer asdsadas eerw dssdfsselleds
Drug causes of gynaecomastia
• spironolactone (most common drug cause)he
• cimetidinehe

• digoxinhe
• cannabishe

• oestrogens, anabolic steroidshe


he earaer aeraer asdsadas eerw dssdfsselleds
Very rare drug causes of gynaecomastia
• tricyclics he
• isoniazidhe
• calcium channel blockershe

• heroinhe
• busulfanhe

• methyldopahe
Which one of the following features of haemochromatosis may be reversible with
treatment?ia

x A.A Cardiomyopathyia
B.A Hypogonadotrophic hypogonadismia
C.A Diabetes mellitusia
D.A Arthropathyia
E.A Liver cirrhosisia
Next question
Haemochromatosis: features
sqweqwesf erwrewfsdfs adasd dhe
Haemochromatosis is an autosomal recessive disorder of iron absorption and metabolism
resulting in iron accumulation. It is caused by inheritance of mutations in the HFE gene
on both copies of chromosome 6*. It is often asymptomatic in early disease and initial
symptoms often non-specific e.g. lethargy and arthralgia
he earaer aeraer asdsadas eerw dssdfsselleds
Presenting features
• 'bronze' skin pigmentationhe
• diabetes mellitushe
• liver: stigmata of chronic liver disease, hepatomegaly, cirrhosis, hepatocellular
deposition)he
• cardiac failure (2nd to dilated cardiomyopathy)he
• hypogonadism (2nd to cirrhosis and pituitary dysfunction - hypogonadotrophic
hypogonadism)he
he earaer aeraer asdsadas eerw dssdfsselleds
Questions have previously been asked regarding which features are reversible with
treatment:
he earaer aeraer asdsadas eerw dssdfsselleds
Reversible complications Irreversible complications
• Cardiomyopathyhe • Liver cirrhosishe
• Skin pigmentationhe • Diabetes mellitushe
• Hypogonadotrophic hypogonadismhe

• Arthropathy

he
he earaer aeraer asdsadas eerw dssdfsselleds
*there are rare cases of families with classic features of genetic haemochromatosis but no
mutation in the HFE gene

Which one of the following is not associated with villous atrophy on jejunal biopsy?ia

A.A Tropical sprueia


B.A Coeliac diseaseia
C.A Hypogammaglobulinaemiaia
x D.A Familial Mediterranean Fever
E.A Whipple's diseaseia
Next question
Causes of villous atrophy (other than coeliacs): tropical sprue, Whipple's, lymphoma,
hypogammaglobulinaemia
Jejunal villous atrophy
sqweqwesf erwrewfsdfs adasd dhe
Whilst coeliac disease is the classic cause of jejunal villous atrophy there are a number of
other causes you need to be aware of
he earaer aeraer asdsadas eerw dssdfsselleds
Causes
• coeliac diseasehe
• tropical spruehe

• hypogammaglobulinaemiahe
• gastrointestinal lymphomahe

• Whipple's diseasehe
Congenital adrenal hyperplasia is most commonly caused by:ia

A.A 11-hydroxylase deficiencyia


B.A Anti-histone antibodiesia
C.A 17-hydroxylase deficiencyia
D.A Anti-mitochondrial antibodiesia
x E.A 21-hydroxylase deficiency
Next question
Congenital adrenal hyperplasia
sqweqwesf erwrewfsdfs adasd dhe
Overview
• group of autosomal recessive disordershe
• affect adrenal steroid biosynthesishe
• in response to resultant low cortisol levels the anterior pituitary secretes high levels
of ACTHhe
• ACTH stimulates the production of adrenal androgens that may virilize a female
infanthe
he earaer aeraer asdsadas eerw dssdfsselleds
Cause
• 21-hydroxylase deficiency (90%)he
• 11-beta hydroxylase deficiency (5%)he
• 17-hydroxylase deficiency (very rare)he
A 22-year-old female presents with recurrent painful oral ulceration. Examination reveals
signs of oral candidal infection. Which one of the following would most suggest type 1
polyglandular syndrome?ia

x A.A Hypocalcaemiaia
B.A Rheumatoid arthritisia
C.A Type II diabetes mellitusia
D.A Coeliac diseaseia
E.A Hypercalcaemiaia
Next question
Primary hypoparathyroidism is usually the first endocrine manifestation of type 1
autoimmune polyendocrinopathy syndrome. The contrast to multiple endocrine neoplasia
(MEN), where hyperparathyroidism is a common finding, should be noted
he earaer aeraer asdsadas eerw dssdfsselleds
The question gives a slightly atypical history as this is the upper end of the age range in
which patients would be expected to present
Autoimmune polyendocrinopathy syndrome
sqweqwesf erwrewfsdfs adasd dhe
Addison's disease (autoimmune hypoadrenalism) is associated with other endocrine
deficiencies in approximately 10% of patients. There are two distinct types of
autoimmune polyendocrinopathy syndrome (APS), with type 2 (sometimes referred to as
Schmidt's syndrome) being much more common.
he earaer aeraer asdsadas eerw dssdfsselleds
APS type 2 has a polygenic inheritance and is linked to HLA DR3/DR4. Patients have
Addison's disease plus either:
• type 1 diabetes mellitushe
• autoimmune thyroid diseasehe
he earaer aeraer asdsadas eerw dssdfsselleds
APS type 1 is occasionally referred to as Multiple Endocrine Deficiency Autoimmune
Candidiasis (MEDAC). It is a very rare autosomal recessive disorder caused by mutation
of AIRE1 gene on chromosome 21
he earaer aeraer asdsadas eerw dssdfsselleds
Features of APS type 1 (2 out of 3 needed)
• chronic mucocutaneous candidiasis (typically first feature as young child)he
• Addison's diseasehe
• primary hypoparathyroidismhe
he earaer aeraer asdsadas eerw dssdfsselleds
Vitiligo can occur in both types
Each one of the following is a feature of polycystic ovarian syndrome, except:ia

A.A Hirsuitismia
B.A Infertilityia
C.A Acneia
D.A Acanthosis nigricansia
x E.A Raised FSH/LH ratio
Next question
A raised LH to FSH ratio is a typical finding in polycystic ovarian syndrome
Polycystic ovarian syndrome
sqweqwesf erwrewfsdfs adasd dhe
Features
• hirsuitism, acnehe
• amenorrhoea, infertilityhe
• obesityhe
• acanthosis nigricanshe
• hypertensionhe

he earaer aeraer asdsadas eerw dssdfsselleds


Investigations
• transvaginal ultrasound of the pelvishe
• impaired glucose tolerancehe
• raised LH/FSH ratiohe
• raised prolactinhe

• mildly raised testosterone - however, if markedly raised consider other causeshe


• positive (withdrawal bleed) response to Provera challengehe
he earaer aeraer asdsadas eerw dssdfsselleds
Management
• metformin: improves both insulin sensitivity and increases chances of conceptionhe
Which one of the following types of thyroid cancer is associated with Hashimoto's
thyroiditis?ia

A.A Anaplasticia
x B.A Lymphomaia
C.A Medullaryia
D.A Follicularia
E.A Papillaryia
Next question
Thyroid cancer
sqweqwesf erwrewfsdfs adasd dhe
Features of hyperthyroidism or hypothyroidism are not commonly seen in patients with
thyroid malignancies as they rarely secrete thyroid hormones
he earaer aeraer asdsadas eerw dssdfsselleds
Type Percentage
Papillary 70% Often young females - excellent prognosis
Follicular 20%
Medullary 5% Cancer of parafollicular cells, secrete calcitonin, part
of MEN-2
Anaplastic 1% Not responsive to treatment, can cause pressure
symptoms
Lymphoma Rare Associated with Hashimoto's

he earaer aeraer asdsadas eerw dssdfsselleds


Management of papillary and follicular cancer
• total thyroidectomyhe

• followed by radioiodine (I-131) to kill residual cellshe


• yearly thyroglobulin levels to detect early recurrent diseasehe
A 54-year-old man with type 2 diabetes mellitus is found on annual review to have new
vessel formation at the optic disc. Visual acuity in both eyes is not affected (6/9). Blood
pressure is 155/84 mmHg.
he earaer aeraer asdsadas eerw dssdfsselleds
HbA1c 8.4%

he earaer aeraer asdsadas eerw dssdfsselleds


What is the most appropriate management?ia

A.A Follow-up ophthalmoscopy in 3 monthsia


B.A Add aspirinia
C.A Blood pressure controlia
D.A Tight glycaemic controlia
x E.A Laser therapyia
Next question
This patient has proliferative diabetic retinopathy and urgent referral to an
ophthalmologist for panretinal photocoagulation is indicated
Diabetic retinopathy
sqweqwesf erwrewfsdfs adasd dhe
Diabetic retinopathy is the most common cause of blindness in adults aged 35-65 years-
old. Hyperglycaemia is thought to cause increased retinal blood flow and abnormal
metabolism in the retinal vessel walls. This precipitates damage to endothelial cells and
pericytes
he earaer aeraer asdsadas eerw dssdfsselleds
Endothelial dysfunction leads to increased vascular permeability which causes the
characteristic exudates seen on fundoscopy. Pericyte dysfunction predisposes to the
formation of microaneurysms. Neovasculization is thought to be caused by the
production of growth factors in response to retinal ischaemia
he earaer aeraer asdsadas eerw dssdfsselleds
In the part 1 exam you are most likely to be asked about the characteristic features of the
various stages/types of diabetic retinopathy
he earaer aeraer asdsadas eerw dssdfsselleds
Background retinopathy
• microaneurysms (dots)he
• blot haemorrhages (<=3)he
• hard exudateshe
he earaer aeraer asdsadas eerw dssdfsselleds
Pre-proliferative retinopathy
• cotton wool spots (soft exudates; ischaemic nerve fibres)he
•>3 blot haemorrhageshe
• venous beading/loopinghe
• deep/dark cluster haemorrhageshe
• more common in Type I DM, treat with laser photocoagulationhe
he earaer aeraer asdsadas eerw dssdfsselleds
Proliferative retinopathy
• retinal neovascularisation - may lead to vitrous haemorrhagehe
• fibrous tissue forming anterior to retinal dische
• more common in Type I DM, 50% blind in 5 yearshe
he earaer aeraer asdsadas eerw dssdfsselleds
Maculopathy
• based on location rather than severity, anything is potentially serioushe
• hard exudates and other 'background' changes on maculahe
• check visual acuityhe

• more common in Type II DMhe


Which one of the following is not an indication for treating a patient with subclinical
hypothyroidism?ia

A.A Previous treatment of Graves' diseaseia


B.A TSH > 10ia
x C.A Raised ESRia
D.A Positive thyroid autoantibodiesia
E.A Other autoimmune disorderia
Subclinical hypothyroidism
sqweqwesf erwrewfsdfs adasd dhe
Basics
• TSH raised but T3, T4 normalhe
• no obvious symptomshe
he earaer aeraer asdsadas eerw dssdfsselleds
Significance
• risk of progressing to overt hypothyroidism is 2-5% per year (higher in men)he
• risk increased by presence of thyroid autoantibodieshe
he earaer aeraer asdsadas eerw dssdfsselleds
Treat if
• TSH > 10he
• thyroid autoantibodies positivehe
• other autoimmune disorderhe
• previous treatment of Graves' diseasehe
A 45-year-old man with a history of alcohol excess is diagnosed as having grade 3
oesophageal varices on endoscopy. What is the most appropriate management to prevent
variceal bleeding?ia

x A.A Propranololia
B.A Isosorbide mononitrateia
C.A Variceal sclerotherapyia
D.A Terlipressinia
E.A Variceal bandingia
Next question
Oesophageal varices
sqweqwesf erwrewfsdfs adasd dhe
Primary prevention
• propranolol - the best current treatmenthe

• nitrateshe
• endoscopic banding for large variceshe
• note sclerotherapy has no real role in primary preventionhe
he earaer aeraer asdsadas eerw dssdfsselleds
Treatment of variceal haemorrhage
• ABChe

• correct clotting: FFP, vitamin Khe


• IV octreotide or terlipressinhe
• endoscopy: banding is superior to sclerotherapyhe
• Sengstaken-Blakemore tube if uncontrolled haemorrhagehe
• Transjugular Intrahepatic Portosystemic Shunt (TIPS) if above measures failhe
A 29-year-old female who is 14 weeks into her first pregnancy is investigated for
excessive sweating and tremor. Blood tests reveal the following:
he earaer aeraer asdsadas eerw dssdfsselleds
TSH < 0.05 mu/l
T4 188 nmol/l

he earaer aeraer asdsadas eerw dssdfsselleds


What is the most appropriate management?ia

A.A Immediate surgeryia


B.A Carbimazoleia
C.A Surgery at start of third trimesteria
x D.A Propylthiouracilia
E.A Radioiodineia
Next question
Propylthiouracil is traditionally taught as the antithyroid drug of choice in pregnancy. It
also has the advantage of being excreted to a lesser extent than carbimazole in breast
milk. However, many endocrinologists will use carbimazole and the BNF states both
drugs may be used in pregnancy. Carbimazole has rarely been associated with aplasia
cutis of the neonate
Pregnancy: thyroid problems
sqweqwesf erwrewfsdfs adasd dhe
In pregnancy there is an increase in the levels of thyroxine-binding globulin (TBG). This
causes an increase in the levels of total thyroxine but does not affect the free thyroxine
level
he earaer aeraer asdsadas eerw dssdfsselleds
Thyrotoxicosis may be due to
• Grave's diseasehe
• activationof TSH receptor by HCG (note HCG levels will fall in second and third
trimester)he
he earaer aeraer asdsadas eerw dssdfsselleds
Management
• traditionally taught that propylthiouracil is the drug of choice, rather than
carbimazolehe
• block-and-replace regimes should not be used in pregnancyhe
• radioiodine therapy is contraindicatedhe
A 64-year-old patient is prescribed pegvisomant for the treatment of acromegaly. What is
the mechanism of action of pegvisomant?ia

A.A IGF-1 receptor antagonistia


x B.A Growth hormone receptor antagonist
C.A IGF-1 receptor agonistia
D.A Growth hormone receptor agonistia
E.A Long-acting somatostatin analogueia
Next question
Acromegaly: management
sqweqwesf erwrewfsdfs adasd dhe
Treatment
• trans-sphenoidal surgery is the treatment of choicehe
• octreotide, a somatostatin analogue (inhibits GH secretion) - a response is seen in >
90% of patientshe
• bromocriptine, a dopamine agonist, often used as adjunct (less effective than
octreotide)he
• pegvisomant (see below)he
• external irradiation
is sometimes used for older patients or following failed
surgical/medical treatmenthe
he earaer aeraer asdsadas eerw dssdfsselleds
Pegvisomant
• highly selective growth hormone receptor antagonisthe
• once daily s/c administrationhe

• decreases IGF-1 levels in 90% of patients to normalhe


• doesn't reduce tumour volume therefore surgery still needed if mass effecthe
Each of the following may complicate hepatitis B infection, except:ia
A.A Glomerulonephritisia
B.A Hepatocellular carcinomaia
x C.A Acute pancreatitis
D.A Chronic infectionia
E.A Polyarteritis nodosaia
Next question
Deterioration in patient with hepatitis B - ? hepatocellular carcinoma
Hepatitis B
sqweqwesf erwrewfsdfs adasd dhe
Basics
• incubation period (weeks) = 10-15he
• double-stranded DNA virushe
• spread: body fluids, verticalhe
• chronic disease in 5-10%he
• vaccination availablehe
he earaer aeraer asdsadas eerw dssdfsselleds
Complications
• chronic infection (5-10%)he

• fulminant liver failure (1%)he


• hepatocellular carcinomahe
• glomerulonephritishe
• polyarteritis nodosahe
• cryoglobulinaemiahe

he earaer aeraer asdsadas eerw dssdfsselleds


Interferon-alpha
• reduces viral replication in 40% of chronic carriershe
• better response to treatment if female, < 50 years old, low HBV DNA levels, non-
Asian, HIV negative, high degree of inflammation on liver biopsyhe
A 43-year-old man is found to have a phaeochromocytoma. Which anti-hypertensive
medication should be started first?ia

A.A Propranololia

B.A Ramiprilia
C.A Atenololia
x D.A Phenoxybenzamineia
E.A Doxazosinia
Next question
Phenoxybenzamine is a non-selective alpha-adrenoceptor antagonist and should be
started before a beta-blocker is introduced
he earaer aeraer asdsadas eerw dssdfsselleds
There is ongoing debate about the optimal medical management of phaeochromocytoma,
with the suggestion that antihypertensive treatment regimes other than nonspecific alpha-
blockade are just as effective and safe. There are however no trials to provide an answer
to this question yet
Phaeochromocytoma
sqweqwesf erwrewfsdfs adasd dhe
Phaeochromocytoma is a rare catecholamine secreting tumour. About 10% are familial
and may be associated with MEN type II, neurofibromatosis and von Hippel-Lindau
syndrome
he earaer aeraer asdsadas eerw dssdfsselleds
Basics
• bilateral in 10%he

• malignant in 10%he
• extra-adrenal in 10% (most common site = organ of Zuckerkandl, adjacent to the
bifurcation of the aorta)he
he earaer aeraer asdsadas eerw dssdfsselleds
Tests
• 24 hr urinary collection of catecholamineshe
he earaer aeraer asdsadas eerw dssdfsselleds
Management
• alpha-blocker (e.g. phenoxybenzamine), given before ahe
• beta -blocker (e.g. propranolol)he
Which one of the following is least likely to cause malabsorption?ia

A.A Systemic sclerosisia


B.A Cystic fibrosisia
C.A Primary biliary cirrhosisia
D.A Whipple's diseaseia
x E.A Haemochromatosisia
Next question
Malabsorption
sqweqwesf erwrewfsdfs adasd dhe
Malabsorption is characterised by diarrhoea, steatorrhoea and weight loss. Causes may be
broadly divided into intestinal (e.g. villous atrophy), pancreatic (deficiency of pancreatic
enzyme production or secretion) and biliary (deficiency of bile-salts needed for
emulsification of fats)
he earaer aeraer asdsadas eerw dssdfsselleds
Intestinal causes of malabsorption
• coeliac diseasehe

• Crohn's diseasehe
• tropical spruehe
• Whipple's diseasehe
• Giardiasishe

• brush border enzyme deficiencies (e.g. lactase insufficiency)he


he earaer aeraer asdsadas eerw dssdfsselleds
Pancreatic causes of malabsorption
• chronic pancreatitishe

• cystic fibrosishe
• pancreatic cancerhe

he earaer aeraer asdsadas eerw dssdfsselleds


Biliary causes of malabsorption
• biliary obstructionhe
• primary biliary cirrhosishe
he earaer aeraer asdsadas eerw dssdfsselleds
Other causes
• bacterial overgrowth (e.g. systemic sclerosis, diverticulae, blind loop)he
• short bowel syndromehe

• lymphomahe
A 45-year-old man is reviewed in the diabetic clinic. His current medication is basal
bolus insulin regime combined with ramipril 10mg od, simvastatin 40mg on and aspirin
75mg od. On examination blood pressure is 122/64 mmHg. The following results are
obtained:araer aeraer asdsadas eerw dssdfsselleds
HbA1c 7.3%
Albumin:Creatinine ratio 10.5

he earaer aeraer asdsadas eerw dssdfsselleds


What is the most appropriate change to his treatment?ia

A.A Stop aspirinia


B.A Increase dose of ramiprilia
x C.A Add an angiotensin-II receptor antagonist
D.A Add doxazosinia
E.A Stop ramiprilia
Next question
The patient is on the maximum daily dose of ramipril. Given the persistently raised
albumin:creatinine ratio dual blockade with an angiotensin-II receptor antagonist is
indicated
Diabetic nephropathy: management
sqweqwesf erwrewfsdfs adasd dhe
Screening
• all patients should be screened annuallyhe
• albumin:creatinine ratio (ACR) in early morning specimenhe
• ACR > 2.5 = microalbuminaemiahe
he earaer aeraer asdsadas eerw dssdfsselleds
Management
• dietary protein restrictionhe
• tight glycaemic controlhe

• BP control: aim for < 125/75 mmHghe


• benefits independent of blood pressure control have been demonstrated for ACE
inhibitors and angiotensin II receptor blockers - these may be used alone or in
combinationhe
• control dyslipidaemia e.g. statinshe
Which one of the following is the most common cause of Cushing's syndrome?ia

A.A Ectopic ACTH productionia


B.A Adrenal adenomaia
C.A Micronodular adrenal dysplasiaia
D.A Adrenal carcinomaia
x E.A Pituitary tumour
Next question
Cushing's disease is the most common, non-iatrogenic, cause of Cushing's syndrome
Cushing's syndrome: causes
sqweqwesf erwrewfsdfs adasd dhe
ACTH dependent causes
• Cushing'sdisease (80%): pituitary tumour secreting ACTH producing adrenal
hyperplasiahe
• ectopic ACTH production (5-10%): e.g. small cell lung cancerhe
he earaer aeraer asdsadas eerw dssdfsselleds
ACTH independent causes
• iatrogenic: steroidshe

• adrenal adenoma (5-10%)he


• adrenal carcinoma (rare)he

• Carney complex: syndrome including cardiac myxomahe


• micronodular adrenal dysplasia (very rare)he
he earaer aeraer asdsadas eerw dssdfsselleds
Pseudo-Cushing's
• mimics Cushing'she
• often due to alcohol excess or severe depressionhe
• causes false positive dexamethasone suppression test or 24 hr urinary free cortisolhe
• insulin stress test may be used to differentiatehe
The first-line treatment in remnant hyperlipidaemia is:ia

A.A Ursodeoxycholic acidia


B.A Vitamin Aia
C.A Statinsia
D.A Fish oilia
x E.A Fibratesia
Next question
Remnant hyperlipidaemia
sqweqwesf erwrewfsdfs adasd dhe
Overview
• rare cause of mixed hyperlipidaemia (raised cholesterol and triglyceride levels)he
• also known as Fredrickson type III hyperlipidaemia, broad-beta disease and
dysbetalipoproteinaemiahe
• associated with apo-e2 homozygosityhe
• high incidence of ischaemic heart disease and peripheral vascular diseasehe
• thought to be caused by impaired removal of IDL from the circulation by the liverhe
he earaer aeraer asdsadas eerw dssdfsselleds
Features
• yellow palmar creaseshe
• palmer xanthomashe
• tuberous xanthomashe
he earaer aeraer asdsadas eerw dssdfsselleds
Management
• fibrates are first line treatmenthe
Which one of the following features is not seen in carcinoid syndrome?ia

A.A Flushingia
B.A Diarrhoeaia
C.A Bronchospasmia
x D.A Hypertension
E.A Pellagraia
Next question
Flushing, diarrhoea, bronchospasm, tricuspid stenosis, pellagra --> carcinoid with liver
mets - diagnosis: urinary 5-HIAA
Hypo- not hypertension is seen in carcinoid syndrome secondary to serotonin release
Carcinoid tumours
sqweqwesf erwrewfsdfs adasd dhe
Carcinoid syndrome
• usuallyoccurs when metastases are present in the liver and release serotonin into the
systemic circulationhe
• may also occur with lung carcinoid as mediators are not 'cleared' by the liverhe
he earaer aeraer asdsadas eerw dssdfsselleds
Features
• diarrhoeahe
• flushinghe

• bronchospasmhe
• hypotensionhe
• right heart valvular stenosis (left heart can be affected in bronchial carcinoid)he
• othermolecules such as ACTH and GHRH may also be secreted resulting in, for
example, Cushing's syndromehe
• pellagra can rarely develop as dietary tryptophan is diverted to serotonin by the
tumourhe
he earaer aeraer asdsadas eerw dssdfsselleds
Investigation
• urinary 5-HIAAhe
• plasma chromogranin A yhe
he earaer aeraer asdsadas eerw dssdfsselleds
Management
• somatostatin analogues e.g. octreotidehe
• diarrhoea: cyproheptadine may helphe

Which one of the following skin disorders is least associated with hypothyroidism?ia

A.A Xanthomataia
B.A Pruritusia
x C.A Pretibial myxoedema
D.A Eczemaia
E.A Dry, coarse hairia
Next question
For the purposes of the MRCP pretibial myxoedema is associated with thyrotoxicosis.
There are however case reports of it been found in hypothyroid patients, especially the
diffuse non-pitting variety
Skin disorders associated with thyroid disease
sqweqwesf erwrewfsdfs adasd dhe
Skin manifestations of hypothyroidism
• dry (anhydrosis), cold, yellowish skinhe
• non-pitting oedema (e.g. hands, face)he
• dry, coarse scalp hair, loss of lateral aspect of eyebrowshe
• eczemahe
• xanthomatahe
he earaer aeraer asdsadas eerw dssdfsselleds
Skin manifestations of hyperthyroidism
• pretibial myxoedema: erythematous, oedematous lesions above the lateral
malleolihe
• thyroid acropachy: clubbinghe
• scalp hair thinninghe
• increased sweatinghe
he earaer aeraer asdsadas eerw dssdfsselleds
Pruritus can occur in both hyper- and hypothyroidism
Which one of the following features is least commonly seen in Gitelman’s syndrome?ia

A.A Hypokalaemiaia
x B.A Hypertensionia
C.A Metabolic alkalosisia
D.A Hypocalciuriaia
E.A Hypomagnesaemiaia
Next question
Gitelman’s syndrome: normotension with hypokalaemia
Gitelman’s syndrome
sqweqwesf erwrewfsdfs adasd dhe
Overview
+
• defect in the thiazide-sensitive Na Cl- transporter in the distal convoluted tubulehe
he earaer aeraer asdsadas eerw dssdfsselleds
Features
• hypokalaemiahe
• hypomagnesaemiahe
• hypocalciuriahe

• metabolic alkalosishe
• normotensionhe

Each one of the following is a feature of metabolic syndrome, except:ia

A.A Impaired glucose toleranceia


B.A High LDLia
x C.A Obstructive sleep apnoea
D.A Central obesityia
E.A Hypertensionia
Next question
Metabolic syndrome
sqweqwesf erwrewfsdfs adasd dhe
Features
• central obesityhe

• hypertensionhe
• impaired glucose tolerancehe
• high fibrinogenhe

• high LDLhe
A 23-year-old male develops type 2 diabetes mellitus. Which one of the following genes
is most likely to be responsible?ia

A.A Glucokinaseia
x B.A HNF-1 alpha
C.A HNF-4 alphaia
D.A HNF-1 betaia
E.A IPF-1ia
Next question
MODY
sqweqwesf erwrewfsdfs adasd dhe
Maturity-onset diabetes of the young (MODY) is characterised by the development of
type 2 diabetes mellitus in patients < 25 years old. It is typically inherited as an
autosomal dominant condition. Over six different genetic mutations have so far been
identified as leading to MODY. Ketosis is not a feature at presentation
he earaer aeraer asdsadas eerw dssdfsselleds
MODY 3
• 60% of caseshe
• due to a defect in the HNF-1 alpha genehe
he earaer aeraer asdsadas eerw dssdfsselleds
MODY 2
• 20% of caseshe
• due to a defect in the glucokinase genehe
What is the most common type of thyroid cancer?ia

A.A Follicularia
B.A Lymphomaia
C.A Medullaryia
D.A Anaplasticia
x E.A Papillaryia
Next question
Thyroid cancer
sqweqwesf erwrewfsdfs adasd dhe
Features of hyperthyroidism or hypothyroidism are not commonly seen in patients with
thyroid malignancies as they rarely secrete thyroid hormones
he earaer aeraer asdsadas eerw dssdfsselleds
Type Percentage
Papillary 70% Often young females - excellent prognosis
Follicular 20%
Medullary 5% Cancer of parafollicular cells, secrete calcitonin, part
of MEN-2
Anaplastic 1% Not responsive to treatment, can cause pressure
symptoms
Lymphoma Rare Associated with Hashimoto's

he earaer aeraer asdsadas eerw dssdfsselleds


Management of papillary and follicular cancer
• total thyroidectomyhe
• followed by radioiodine (I-131) to kill residual cellshe
• yearly thyroglobulin levels to detect early recurrent diseasehe
A 54 year female consults her GP concerned about double vision. She is noted to have
exophthalmos, proptosis and conjunctival oedema on examination and a diagnosis of
thyroid eye disease is suspected. What can be said regarding her thyroid status?ia

A.A Hyper- or euthyroidia


B.A Hypothyroidia
C.A Hyperthyroidia
D.A Hypo- or euthyroidia
x E.A Eu-, hypo- or hyperthyroid
Next question
Thyroid eye disease
sqweqwesf erwrewfsdfs adasd dhe
Thyroid eye disease is a clinical diagnosis based on the features described below. It is due
by thyroid autoantibodies causing retro-orbital inflammation, although these may not
necessarily be detectable. The patient may be eu-, hypo- or hyperthyroid at the time of
presentation
he earaer aeraer asdsadas eerw dssdfsselleds
Features
• exophthalmoshe
• proptosishe
• conjunctival oedemahe

• papilloedemahe

• ophthalmoplegiahe

he earaer aeraer asdsadas eerw dssdfsselleds


Smoking increases risk of patients with Grave's disease developing eye disease
Each of the following statements regarding hyperkalaemia are true, except:ia

x A.A ECG changes include large P waves


B.A It is associated with metabolic acidosisia
C.A May be caused by Addison's diseaseia
D.A May be caused by spironolactoneia
E.A May be caused by massive blood transfusionia
Next question
Hyperkalaemia
sqweqwesf erwrewfsdfs adasd dhe
Plasma potassium levels are regulated by a number of factors including aldosterone, acid-
base balance and insulin levels. Metabolic acidosis is associated with hyperkalaemia as
hydrogen and potassium ions compete with each other for exchange with sodium ions
across cell membranes and in the distal tubule. ECG changes seen in hyperkalaemia
include tall-tented T waves, small P waves, widened QRS leading to a sinusoidal pattern
and asystole
he earaer aeraer asdsadas eerw dssdfsselleds
Causes of hyperkalaemia:
• acute renal failurehe
• drugs*: potassium sparing diuretics, ACE inhibitors, ciclosporinhe
• metabolic acidosishe
• Addison'she
• rhabdomyolysishe

• massive blood transfusionhe


he earaer aeraer asdsadas eerw dssdfsselleds
*beta-blockers interfere with potassium transport into cells and can potentially cause
hyperkalaemia in renal failure patients - remember beta-agonists, e.g. salbutamol, are
sometimes used as emergency treatment
Which of the following statements is true regarding the pathophysiology of diabetes
mellitus?ia

A.A Concordance between identical twins is higher in type 2 diabetes mellitus


x
than type 1
B.A Patients with type 1 diabetes mellitus are rarely HLA-DR4 positiveia
C.A Type 2 diabetes mellitus is associated with HLA-DR3ia
D.A Haemochromatosis is an example of primary diabetesia
E.A Type 1 diabetes mellitus is thought to be inherited in an autosomal
dominant fashionia
Next question
Type 1 diabetes mellitus is caused by autoimmune destruction of the Beta-cells of the
pancreas. Identical twins show a genetic concordance of 40%. It is associated with HLA-
DR3 and DR4. It is inherited in a polygenic fashion
he earaer aeraer asdsadas eerw dssdfsselleds
Type 2 diabetes mellitus is thought to be caused by a relative deficiency of insulin and the
phenomenon of insulin resistance. Age, obesity and ethnicity are important aetiological
factors. There is almost 100% concordance in identical twins and no HLA associations.
he earaer aeraer asdsadas eerw dssdfsselleds
Haemochromatosis is an example of secondary diabetes
Diabetes: pathophysiology
sqweqwesf erwrewfsdfs adasd dhe
Type 1 DM
• autoimmune diseasehe
• antibodies against beta cells of pancreashe
• various antibodies such as islet-associated antigen (IAA) antibody and glutamic acid
decarboxylase (GAD) antibody are detected in patients who later go on to develop
type 1 DM - their prognostic significance is not yet clearhe
What is the mode of inheritance of haemochromatosis?ia

x A.A Autosomal recessive


B.A X-linked dominantia
C.A Mitochondrial inheritanceia
D.A Autosomal dominantia
E.A X-linked recessiveia
Haemochromatosis: features
sqweqwesf erwrewfsdfs adasd dhe
Haemochromatosis is an autosomal recessive disorder of iron absorption and metabolism
resulting in iron accumulation. It is caused by inheritance of mutations in the HFE gene
on both copies of chromosome 6*. It is often asymptomatic in early disease and initial
symptoms often non-specific e.g. lethargy and arthralgia
he earaer aeraer asdsadas eerw dssdfsselleds
Presenting features
• 'bronze' skin pigmentationhe
• diabetes mellitushe
• liver: stigmata of chronic liver disease, hepatomegaly, cirrhosis, hepatocellular
deposition)he
• cardiac failure (2nd to dilated cardiomyopathy)he
• hypogonadism (2nd to cirrhosis and pituitary dysfunction - hypogonadotrophic
hypogonadism)h
er asdsadas eerw dssdfsselleds
Questions have previously been asked regarding which features are reversible with
treatment:r asdsadas eerw dssdfsselleds
Reversible complications Irreversible complications
• Cardiomyopathyhe • Liver cirrhosishe
• Skin pigmentationhe • Diabetes mellitushe
• Hypogonadotrophic hypogonadismhe
• Arthropathy

he
he earaer aeraer asdsadas eerw dssdfsselleds
*there are rare cases of families with classic features of genetic haemochromatosis but no
mutation in the HFE gene
Which one of the following is least characteristic of Addison's disease?ia

A.A Hypoglycaemiaia
x B.A Metabolic alkalosis
C.A Hyponatraemiaia
D.A Hyperkalaemiaia
E.A Positive short ACTH testia
Next question
Addison's disease is associated with a metabolic acidosis
Addison's disease: investigations
sqweqwesf erwrewfsdfs adasd dhe
In a patient with suspect Addison's disease the definite investigation is a short ACTH test.
Plasma cortisol is measure before and 30 minutes after giving Synacthen 250ug IM.
Adrenal autoantibodies such as anti-21-hydroxylase may also be demonstrated
he earaer aeraer asdsadas eerw dssdfsselleds
Associated electrolyte abnormalities
• hyperkalaemiahe
• hyponatraemiahe

• hypoglycaemiahe
• metabolic acidosishe

Each one of the following is a contraindication to performing a percutaneous liver biopsy,


except:ia

A.A INR 2.6ia


x B.A Viral hepatitis
C.A Hydatid cystia
D.A Uncooperative patientia
E.A Haemoangiomaia
Next question
Liver biopsy
sqweqwesf erwrewfsdfs adasd dhe
Contraindications to percutaneous liver biopsy
• deranged clotting (varying cut-offs, e.g. INR > 1.5)he
• low platelets (varying cut-offs, e.g. < 80)he
• anaemiahe
• bile duct obstructionhe
• hydatid cysthe

• haemoangiomahe
• uncooperative patienthe

• asciteshe
What is the most common cause of primary hyperaldosteronism?ia

A.A Pituitary tumouria


B.A Adrenocortical adenomaia
C.A Adrenal carcinomaia
D.A Ectopic secretionia
x E.A Bilateral idiopathic adrenal hyperplasia
Next question
bilateral idiopathic adrenal hyperplasia is the most common cause of primary
hyperaldosteronism
Primary hyperaldosteronism
sqweqwesf erwrewfsdfs adasd dhe
Primary hyperaldosteronism was previously thought to be most commonly caused by an
adrenal adenoma, termed Conn's syndrome. However, recent studies have shown that
bilateral idiopathic adrenal hyperplasia is the cause in up to 70% of cases. Differentiating
between the two is important as this determines treatment. Adrenal carcinoma is an
extremely rare cause of primary hyperaldosteronism
he earaer aeraer asdsadas eerw dssdfsselleds
Features
• hypertensionhe
• hypokalaemia (e.g. muscle weakness)he
• alkalosishe

he earaer aeraer asdsadas eerw dssdfsselleds


Investigations
• high serum aldosteronehe

• low serum reninhe


• high-resolution CT abdomenhe
he earaer aeraer asdsadas eerw dssdfsselleds
Management
• adrenal adenoma: surgeryhe

• bilateral adrenocortical hyperplasia: aldosterone antagonist e.g. spironolactonehe


Which one of the following is least associated with hepatocellular carcinoma?ia
A.A Hepatitis Cia
B.A Primary biliary cirrhosisia
C.A Aflatoxinia
x D.A Wilson's disease
E.A Haemochromatosisia
Next question
Hepatocellular carcinoma
sqweqwesf erwrewfsdfs adasd dhe
Risk factors
• hepatitis B and Che
• cirrhosis*: alcohol, haemochromatosis, primary biliary cirrhosishe
• alpha-1 antitrypsin deficiencyhe
• hereditary tyrosinosishe
• glycogen storage diseasehe
• aflatoxinhe

• drugs: oral contraceptive pill, anabolic steroidshe


• porphyria cutanea tardahe
he earaer aeraer asdsadas eerw dssdfsselleds
*Wilson's disease is an exception
Which one of the following is not a recognised treatment option in acromegaly?ia

A.A Trans-sphenoidal surgeryia


B.A Octreotideia
C.A Bromocriptineia
D.A Pegvisomantia
x E.A Somatotropin
Next question
Somatotropin is used in growth hormone deficiency
Acromegaly: management
sqweqwesf erwrewfsdfs adasd dhe
Treatment
• trans-sphenoidal surgery is the treatment of choicehe
• octreotide, a somatostatin analogue (inhibits GH secretion) - a response is seen in >
90% of patientshe
• bromocriptine, a dopamine agonist, often used as adjunct (less effective than
octreotide)he
• pegvisomant (see below)he
• external irradiation is sometimes used for older patients or following failed
surgical/medical treatmenthe
he earaer aeraer asdsadas eerw dssdfsselleds
Pegvisomant
• highly selective growth hormone receptor antagonisthe
• once daily s/c administrationhe
• decreases IGF-1 levels in 90% of patients to normalhe
• doesn't reduce tumour volume therefore surgery still needed if mass effecthe
Which one of the following causes of delayed puberty is associated with short stature?ia

A.A Klinefelter's syndromeia


x B.A Turner's syndrome
C.A Polycystic ovarian syndromeia
D.A Androgen insensitivityia
E.A Kallman's syndromeia
Next question
Delayed puberty
sqweqwesf erwrewfsdfs adasd dhe
Delayed puberty with short stature
• Turner's syndromehe
• Prader-Willi syndromehe

• Noonan's syndromehe
he earaer aeraer asdsadas eerw dssdfsselleds
Delayed puberty with normal stature
• polycystic ovarian syndromehe
• androgen insensitivityhe
• Kallman's syndromehe
• Klinefelter's syndromehe
Which one of the following is the most cause of hypothyroidism in the UK?ia

A.A Pituitary failureia


B.A Dietary iodine deficiencyia
C.A Lithium therapyia
x D.A Primary atrophic hypothyroidism
E.A Hashimoto's thyroiditisia
Next question
Hypothyroidism
sqweqwesf erwrewfsdfs adasd dhe
Primary hypothyroidism
he earaer aeraer asdsadas eerw dssdfsselleds
Primary atrophic hypothyroidism
• most common causehe

• autoimmune disease, associated with IDDM, Addison's or pernicious anaemiahe


• 6 times more common in womenhe
he earaer aeraer asdsadas eerw dssdfsselleds
Hashimoto's thyroiditis
• autoimmune disease as above with goitre (positive microsomal antibodies)he
• 10 times more common in womenhe
he earaer aeraer asdsadas eerw dssdfsselleds
After thyroidectomy or radioiodine treatment
Drug therapy (e.g. lithium, amiodarone or anti-thyroid drugs such as carbimazole)
Dietary iodine deficiency
he earaer aeraer asdsadas eerw dssdfsselleds
Secondary hypothyroidism (rare)
he earaer aeraer asdsadas eerw dssdfsselleds
From pituitary failure
Dynamic pituitary function tests may be used to assess each one of the following,
except:ia

A.A Cortisolia
B.A Prolactinia
C.A Growth hormoneia
D.A Follicular stimulating hormoneia
x E.A Antidiuretic hormone
Next question
Dynamic pituitary function tests
sqweqwesf erwrewfsdfs adasd dhe
A dynamic pituitary function test is used to assess patients with suspected primary
pituitary dysfunction
he earaer aeraer asdsadas eerw dssdfsselleds
Insulin, TRH and LHRH are given to the patient following which the serum glucose,
cortisol, growth hormone, TSH, LH and FSH levels are recorded at regular intervals.
Prolactin levels are also sometimes measured*
he earaer aeraer asdsadas eerw dssdfsselleds
A normal dynamic pituitary function test has the following characteristics:
• GH level rises > 20mu/lhe
• cortisol level rises > 550 mmol/lhe
• TSH level rises by > 2 mu/l from baseline levelhe
• LH and FSH should doublehe
he earaer aeraer asdsadas eerw dssdfsselleds
*dopamine antagonist tests using metoclopramide may also be used in the investigation
of hyperprolactinaemia. A normal response is at least a twofold rise in prolactin. A
blunted prolactin response suggests a prolactinoma
A 53 year man presents to his GP as his wife has noticed a change in his appearance. He
has also noticed his hands seem larger. On examination blood pressure is 170/94 and he is
noted to have bitemporal hemianopia. What is the definitive treatment?ia

A.A Octreotideia
B.A External irradiationia
C.A Pegvisomantia
x D.A Trans-sphenoidal surgery
E.A Bromocriptineia
Acromegaly: management
sqweqwesf erwrewfsdfs adasd dhe
Treatment
• trans-sphenoidal surgery is the treatment of choicehe
• octreotide, a somatostatin analogue (inhibits GH secretion) - a response is seen in >
90% of patientshe
• bromocriptine, a dopamine agonist, often used as adjunct (less effective than
octreotide)he
• pegvisomant (see below)he

• external irradiation
is sometimes used for older patients or following failed
surgical/medical treatmenthe
he earaer aeraer asdsadas eerw dssdfsselleds
Pegvisomant
• highly selective growth hormone receptor antagonisthe
• once daily s/c administrationhe
• decreases IGF-1 levels in 90% of patients to normalhe
• doesn't reduce tumour volume therefore surgery still needed if mass effecthe
Cushing's syndrome is most typically associated with which one of the following
abnormalities:ia

A.A Hypokalaemic metabolic acidosisia


B.A Hyperkalaemic metabolic alkalosisia
C.A Hypocalcaemic metabolic acidosisia
x D.A Hypokalaemic metabolic alkalosis
E.A Hyperkalaemic metabolic acidosisia
Next question
Cushing's syndrome: investigations
sqweqwesf erwrewfsdfs adasd dhe
Investigations are divided into confirming Cushing's syndrome and then localising the
lesion. A hypokalaemic metabolic alkalosis may be seen, along with impaired glucose
tolerance. Ectopic ACTH secretion (e.g. secondary to small cell lung cancer) is
characteristically associated with very low potassium levels. An insulin stress test is used
to differentiate between true Cushing's and pseudo-Cushing's
he earaer aeraer asdsadas eerw dssdfsselleds
Tests to confirm Cushing's syndrome
he earaer aeraer asdsadas eerw dssdfsselleds
The two most commonly used tests are:
• overnight dexamethasone suppression test (most sensitive)he
• 24 hr urinary free cortisolhe
he earaer aeraer asdsadas eerw dssdfsselleds
Localisation tests
he earaer aeraer asdsadas eerw dssdfsselleds
The first-line localisation is 9am and midnight plasma ACTH (and cortisol) levels. If
ACTH is suppressed then a non-ACTH dependent cause is likely such as an adrenal
adenoma
he earaer aeraer asdsadas eerw dssdfsselleds
High-dose dexamethasone suppression test
• if pituitary source then cortisol suppressedhe
• if ectopic/adrenal then no change in cortisolhe
he earaer aeraer asdsadas eerw dssdfsselleds
CRH stimulation
• if pituitary source then cortisol riseshe
• if ectopic/adrenal then no change in cortisolhe
he earaer aeraer asdsadas eerw dssdfsselleds
Petrosal sinus sampling of ACTH may be needed to differentiate between pituitary and
ectopic ACTH secretion
Each one of the following is associated with Pendred's syndrome, except:ia

A.A Goitreia
x B.A Short 4th and 5th metacarpals
C.A Autosomal recessive inheritanceia
D.A Sensorineural deafnessia
E.A Euthyroid statusia
Next question
Pendred's syndrome
sqweqwesf erwrewfsdfs adasd dhe
Autosomal recessive disorder of defective iodine uptake
he earaer aeraer asdsadas eerw dssdfsselleds
Features
• sensorineural deafnesshe
• goitrehe
• euthyroidhe

Which one of the following is most associated with oesophageal cancer?ia

x A.A Coeliac disease


B.A Hypothyroidismia
C.A Crohn's diseaseia
D.A Addison's diseaseia
E.A Ulcerative colitisia
Next question
Oesophageal cancer
sqweqwesf erwrewfsdfs adasd dhe
Overview
• classically squamouscarcinoma is the most common cell type but the incidence of
adenocarcinoma is rising rapidlyhe
• majority of tumours are in the middle thirdhe
he earaer aeraer asdsadas eerw dssdfsselleds
Risk factors
• smokinghe

• alcoholhe
• GORDhe

• Barrett's oesophagushe
• achalasiahe
• Plummer-Vinson syndromehe
• rare: coeliac disease, sclerodermahe
he earaer aeraer asdsadas eerw dssdfsselleds
Management
• CT for staginghe
What is the most appropriate screening investigation to exclude a phaeochromocytoma?ia

A.A Ultrasound adrenalsia


B.A Phenoxybenzamine suppression testia
C.A 24 hour urinary collection of vanillylmandelic acidia
x D.A 24 hour urinary collection of catecholamines
E.A Plasma adrenaline (morning)ia
Next question
Phaeochromocytoma: do 24 hr urinary catecholamines, not VMA etc
A 24 hr urinary collection of catecholamines is preferred to one of vanillylmandelic acid
as it has a higher sensitivity. Three 24 hour collections are needed as some patients have
intermittently raised levels
Phaeochromocytoma
sqweqwesf erwrewfsdfs adasd dhe
Phaeochromocytoma is a rare catecholamine secreting tumour. About 10% are familial
and may be associated with MEN type II, neurofibromatosis and von Hippel-Lindau
syndrome
he earaer aeraer asdsadas eerw dssdfsselleds
Basics
• bilateral in 10%he
• malignant in 10%he
• extra-adrenal in 10% (most common site = organ of Zuckerkandl, adjacent to the
bifurcation of the aorta)he
he earaer aeraer asdsadas eerw dssdfsselleds
Tests
• 24 hr urinary collection of catecholamineshe
he earaer aeraer asdsadas eerw dssdfsselleds
Management
• alpha-blocker (e.g. phenoxybenzamine), given before ahe
• beta -blocker (e.g. propranolol)he
A 54-year-old man with type 2 diabetes mellitus is reviewed in clinic. He is currently
taking rosiglitazone, aspirin and simvastatin. Which one of the following problems is
most likely caused by rosiglitazone?ia

A.A Photosensitivityia
B.A Thrombocytopaeniaia
C.A Myalgiaia
x D.A Peripheral oedema
E.A Hyponatraemiaia
Next question
Thiazolidinediones
sqweqwesf erwrewfsdfs adasd dhe
Thiazolidinediones are a new class of agents used in the treatment of type 2 diabetes
mellitus. They are agonists to the PPAR-gamma receptor and reduce peripheral insulin
resistance
he earaer aeraer asdsadas eerw dssdfsselleds
The PPAR-gamma receptor is an intracellular nuclear receptor. Its natural ligands are free
fatty acids and it is thought to control adipocyte differentiation and function
he earaer aeraer asdsadas eerw dssdfsselleds
Adverse effects
• weight gainhe

• liver impairment: monitor LFTshe


• fluid retention - therefore contraindicated in heart failurehe
The following cause a rise in triglyceride levels, as opposed to cholesterol levels,
except:ia

x A.A Hypothyroidismia
B.A Obesityia
C.A Liver diseaseia
D.A Bendrofluazideia
E.A Chronic renal failureia
Hyperlipidaemia: secondary causes
sqweqwesf erwrewfsdfs adasd dhe
Causes of predominately hypertriglyceridaemia
• diabetes mellitus (types 1 and 2)he
• obesityhe
• alcoholhe

• chronic renal failurehe


• drugs: thiazides, non-selective beta-blockers, unopposed oestrogenhe
• liver diseasehe
he earaer aeraer asdsadas eerw dssdfsselleds
Causes of predominately hypercholesterolaemia
• nephrotic syndromehe

• cholestasishe
• hypothyroidismhe

Which one of the following is not a recognised dermatological manifestations of


hyperthyroidism?ia

A.A Pruritusia
B.A Pretibial myxoedemaia
C.A Increased sweatingia
x D.A Loss of lateral aspect of eyebrows
E.A Thyroid acropachyia
Next question
Skin disorders associated with thyroid disease
sqweqwesf erwrewfsdfs adasd dhe
Skin manifestations of hypothyroidism
• dry (anhydrosis), cold, yellowish skinhe
• non-pitting oedema (e.g. hands, face)he
• dry, coarse scalp hair, loss of lateral aspect of eyebrowshe
• eczemahe
• xanthomatahe
he earaer aeraer asdsadas eerw dssdfsselleds
Skin manifestations of hyperthyroidism
• pretibial myxoedema: erythematous, oedematous lesions above the lateral
malleolihe
• thyroid acropachy: clubbinghe
• scalp hair thinninghe
• increased sweatinghe
he earaer aeraer asdsadas eerw dssdfsselleds
Pruritus can occur in both hyper- and hypothyroidism
Each one of the following is associated with autoimmune polyendocrinopathy syndrome
type 1, except:ia

A.A Chronic mucocutaneous candidiasisia


B.A Addison's diseaseia
x C.A Primary hyperparathyroidism
D.A Autosomal recessive inheritanceia
E.A A mutation of the AIRE1 gene on chromosome 21ia
Next question
Autoimmune polyendocrinopathy syndrome
sqweqwesf erwrewfsdfs adasd dhe
Addison's disease (autoimmune hypoadrenalism) is associated with other endocrine
deficiencies in approximately 10% of patients. There are two distinct types of
autoimmune polyendocrinopathy syndrome (APS), with type 2 (sometimes referred to as
Schmidt's syndrome) being much more common.
he earaer aeraer asdsadas eerw dssdfsselleds
APS type 2 has a polygenic inheritance and is linked to HLA DR3/DR4. Patients have
Addison's disease plus either:
• type 1 diabetes mellitushe
• autoimmune thyroid diseasehe
he earaer aeraer asdsadas eerw dssdfsselleds
APS type 1 is occasionally referred to as Multiple Endocrine Deficiency Autoimmune
Candidiasis (MEDAC). It is a very rare autosomal recessive disorder caused by mutation
of AIRE1 gene on chromosome 21
he earaer aeraer asdsadas eerw dssdfsselleds
Features of APS type 1 (2 out of 3 needed)
• chronic mucocutaneous candidiasis (typically first feature as young child)he
• Addison's diseasehe
• primary hypoparathyroidismhe

he earaer aeraer asdsadas eerw dssdfsselleds


Vitiligo can occur in both types
Each one of the following is a feature of subacute thyroiditis, except:ia
A.A Good prognosisia
x B.A Increased iodine uptake on scan
C.A Painful goitreia
D.A Hyperthyroidismia
E.A Elevated ESRia
Next question
Subacute (De Quervain's) thyroiditis is associated with decreased iodine uptake on scan
Subacute (De Quervain's) thyroiditis
sqweqwesf erwrewfsdfs adasd dhe
Subacute thyroiditis (also known as De Quervain's thyroiditis) is thought to occur
following viral infection and typically presents with hyperthyroidism
he earaer aeraer asdsadas eerw dssdfsselleds
Features
• hyperthyroidismhe
• painful goitrehe

• raised ESRhe
• globally reduced uptake on iodine-131 scanhe
he earaer aeraer asdsadas eerw dssdfsselleds
Management
• usually self-limiting - most patients do not require treatmenthe
• steroids may be used, particularly if hypothyroidism developshe
A 53-year-old heavy goods vehicle driver with a history of type II diabetes mellitus is
reviewed in the diabetes clinic. Despite maximal oral hypoglycaemic therapy his HbA1c
is 9.7%. If insulin therapy is started then which one of the following is most appropriate
with regards to his job?ia

x A.A Cannot continue to drive heavy goods vehicle


B.A Inform DVLA and recommence driving once stable insulin dose
achievedia
C.A Can only drive during daylight hoursia
D.A As under 55 years of age then no requirement to inform DVLAia
E.A Needs annual screening to exclude retinopathy or neuropathyia
Next question
Patients on insulin cannot hold a HGV licence
DVLA: diabetes mellitus
sqweqwesf erwrewfsdfs adasd dhe
The guidelines below relate to car/motorcycle use unless specifically stated. For obvious
reasons, the rules relating to drivers of heavy goods vehicles (HGVs) tend to be much
stricter
he earaer aeraer asdsadas eerw dssdfsselleds
Specific rules
• if on insulin then cannot hold HGV licence*he
• if on insulin then
patient can drive a car as long as they have hypoglycaemic
awareness and no relevant visual impairmenthe
• if diet controlled
alone and no relevant complications (e.g. maculopathy) then no
requirement to inform DVLAhe
he earaer aeraer asdsadas eerw dssdfsselleds
*there are complex exceptions to this rule, but these are not relevant for the purposes of
the exam
Each one of the following is a cause of nephrogenic diabetes insipidus, except:ia

x A.A Hypocalcaemiaia
B.A Sickle-cell anaemiaia
C.A Lithiumia
D.A Hypokalaemiaia
E.A Demeclocyclineia
Next question
Diabetes insipidus
sqweqwesf erwrewfsdfs adasd dhe
Causes of cranial DI
• idiopathiche
• post head injuryhe

• pituitary surgeryhe
• craniopharyngiomashe
• histiocytosis Xhe
he earaer aeraer asdsadas eerw dssdfsselleds
DIDMOAD is the association of cranial Diabetes Insipidus, Diabetes Mellitus, Optic
Atrophy and Deafness (also known as Wolfram's syndrome)
he earaer aeraer asdsadas eerw dssdfsselleds
Causes of nephrogenic DI
• genetic (primary)he
• electrolytes: hypercalcaemia, hypokalaemiahe
• drugs: demeclocycline, lithiumhe
• tubulo-interstitial disease: obstruction, sickle-cell, pyelonephritishe
he earaer aeraer asdsadas eerw dssdfsselleds
Investigation
• high plasma osmolarity, low urine osmolarityhe
• water deprivation testhe
Which one of the following conditions may cause hypokalaemia in association with
hypertension?ia
A.A Gitelman syndromeia
B.A 21-hydroxylase deficiencyia
C.A Bartter's syndromeia
D.A Phaeochromocytomaia
x E.A 11-beta hydroxylase deficiency
Next question
Hypokalaemia and hypertension
sqweqwesf erwrewfsdfs adasd dhe
For the MRCP it is useful to be able to classify the causes of hypokalaemia in to those
associated with hypertension, and those which are not
he earaer aeraer asdsadas eerw dssdfsselleds
Hypokalaemia with hypertension
• Cushing's syndromehe
• Conn's syndrome (primary hyperaldosteronism)he
• Liddle's syndromehe
• congenital adrenal hyperplasia (11-beta hydroxylase deficiency)he

he earaer aeraer asdsadas eerw dssdfsselleds


Carbenoxolone, an anti-ulcer drug, and liquorice excess can potentially cause
hypokalaemia associated with hypertension
he earaer aeraer asdsadas eerw dssdfsselleds
Hypokalaemia without hypertension
• diureticshe
• GI loss (e.g. diarrhoea, vomiting)he
• renal tubular acidosis (type 1 and 2*)he
• Bartter's syndromehe
• Gitelman syndromehe
he earaer aeraer asdsadas eerw dssdfsselleds
Urinary potassium may be helpful in diagnosis, if < 30mmol/day then GI loss or diuretic
use is likely to be the cause
he earaer aeraer asdsadas eerw dssdfsselleds
*type 4 renal tubular acidosis is associated with hyperkalaemia
A 30-year-old female is started on carbimazole 20mg bd following a diagnosis of Grave's
disease. What is the best biochemical marker to assess her response to treatment?ia

A.A Total T4ia


x B.A TSHia
C.A Free T4ia
D.A ESRia
E.A Free T3ia
Next question
The answer the College are looking for is TSH. There is however a significant proportion
of patients for whom TSH monitoring alone is insufficient. TSH may remain suppressed
for several weeks as continued production of thyroid stimulating immunoglobulins seen
in Grave's disease reduces the need for the pituitary to secrete TSH
Thyrotoxicosis
sqweqwesf erwrewfsdfs adasd dhe
Causes
• Grave's diseasehe
• toxic nodule goitreshe
• subacute (de Quervain's) thyroiditishe
• post-partum thyroiditishe

• Hashimoto's thyroiditishe
• toxic adenoma (Plummer's disease)he
• amiodarone therapyhe
he earaer aeraer asdsadas eerw dssdfsselleds
Investigation
• TSH down, T4 and T3 uphe
• thyroid autoantibodieshe
• other investigations are not routinely done but includes isotope scanninghe
What causes increased sweating in patients with acromegaly?i
a

A.A Increased sodium content in sweatia


B.A Decreased LH and FSH secondary to hypopituitarismia
C.A Episodic hypoglycaemiaia
D.A Low-grade chronic pyrexiaia
x E.A Sweat gland hypertrophy
Next question
Acromegaly: features
sqweqwesf erwrewfsdfs adasd dhe
In acromegaly there is excess growth hormone secondary to a pituitary adenoma in over
95% of cases. A minority of cases are caused by ectopic GHRH or GH production by
tumours e.g. pancreatic
he earaer aeraer asdsadas eerw dssdfsselleds
Features
• course, oily skin , large tongue, prognathism, interdental spaceshe
• spade-like hands, increase in shoe sizehe
• features of pituitary tumour: hypopituitarism, headaches, bitemporal hemianopiahe
• raised prolactin in 1/3 of cases --> galactorrhoeahe
• 6% of patients have MEN-1he
he earaer aeraer asdsadas eerw dssdfsselleds
Complications
• hypertensionhe

• diabetes (>10%)he
• cardiomyopathyhe
• colorectal cancerhe

Which of the following results establishes a diagnosis of impaired fasting glucose?ia

A.A Fasting glucose 7.1 mmol/L on one occasionia


x B.A Fasting glucose 6.8 mmol/L on two occasions
C.A Glycosuria ++ia
D.A 75g oral glucose tolerance test 2 hour value of 8.4 mmol/Lia
E.A HbA1c of 6.7%ia
Next question
Diabetes diagnosis: fasting > 7.0, random > 11.1 - if asymptomatic need two readings
A 75g oral glucose tolerance test 2 hour value of 8.4 mmol/L would imply impaired
glucose tolerance rather than impaired fasting glucose
Diabetes mellitus: diagnosis
sqweqwesf erwrewfsdfs adasd dhe
Based on 1997 American Diabetes Association (ADA) guidelines
he earaer aeraer asdsadas eerw dssdfsselleds
If symptomatic
• fasting glucose > 7.0 mmol/lhe
• random glucose > 11.1 mmol/l (or after 75g oral glucose tolerance test)he
he earaer aeraer asdsadas eerw dssdfsselleds
If asymptomatic above criteria apply but must be repeated on two occasions
he earaer aeraer asdsadas eerw dssdfsselleds
Impaired fasting glucose and impaired glucose tolerance
he earaer aeraer asdsadas eerw dssdfsselleds
A fasting glucose >= 6.1 but < 7.0 mmol/l implies impaired fasting glucose (IFG)
he earaer aeraer asdsadas eerw dssdfsselleds
Impaired glucose tolerance (IGT) is defined as fasting plasma glucose < 7.0 mmol/l and
OGTT 2-hour value >= 7.8 mmol/l but < 11.1 mmol/l
he earaer aeraer asdsadas eerw dssdfsselleds
Diabetes UK suggests:
• 'People with IFG should then be offered an oral glucose tolerance test to rule out a
diagnosis of diabetes. A result below 11.1 mmol/l but above 7.8 mmol/l indicates
that the person doesn’t have diabetes but does have IGT.'he

A 62-year-old male with a history of type 2 diabetes mellitus is seen in the pre-op clinic
prior to a left total knee replacement. Routine blood tests are as follows:
he earaer aeraer asdsadas eerw dssdfsselleds
Na+ 139 mmol/l
K+ 4.2 mmol/l
Bicarbonate 15 mmol/l
Urea 15.2 mmol/l
Creatinine 267 µmol/l
Glucose 9.2 mmol/l
he earaer aeraer asdsadas eerw dssdfsselleds
What is the likely cause of the bicarbonate value?ia

A.A Vomiting due to gastroparesisia


B.A Renal tubular acidosisia
C.A Addison's diseaseia
x D.A Metforminia
E.A Rosiglitazoneia
Next question
Whilst the decreased bicarbonate value may be contributed to by deteriorating renal
function, it is important to exclude lactic acidosis secondary to metformin
Metformin
sqweqwesf erwrewfsdfs adasd dhe
Metformin is a biguanide used mainly in the treatment of type 2 diabetes mellitus. It has a
number of actions which improves glucose tolerance (see below). Unlike sulphonylureas
it does not cause hypoglycaemia and weight gain and is therefore first-line if the patient
is overweight. Another use of metformin is in polycystic ovarian syndrome where it has
been shown to increase ovulation and reduce hirsuitism
he earaer aeraer asdsadas eerw dssdfsselleds
Mechanism of action
• increases insulin sensitivityhe
• decreases hepatic gluconeogenesishe
• may also reduce gastrointestinal absorption of carbohydrateshe
he earaer aeraer asdsadas eerw dssdfsselleds
Adverse effects
• gastrointestinal upsets are common (nausea, anorexia, diarrhoea), intolerable in
20%he
• reduced vitamin B12 absorption - rarely a clinical problemhe
he earaer aeraer asdsadas eerw dssdfsselleds
Contraindications
• lactic acidosis with severe liver disease / renal failure / heart failurehe
• do not use during suspected episodes of tissue hypoxia (e.g. recent MI, sepsis)he
• alcohol abuse is a relative contraindicationhe
• pregnancyhe

• stop 2 days before GA, restart when renal function normalhe


• stop prior to IV contrast e.g. angiography, restart when renal function normalhe
Which one of the following features is least associated with primary
hyperparathyroidism?ia

A.A Depressionia
B.A Polydipsiaia
x C.A Sensory loss
D.A Peptic ulcerationia
E.A Hypertensionia
Next question
Primary hyperparathyroidism
sqweqwesf erwrewfsdfs adasd dhe
In the MRCP, primary hyperparathyroidism is stereotypically seen in elderly females with
an unquenchable thirst and an inappropriately normal or raised parathyroid hormone
level. It is most commonly due to a solitary adenoma
he earaer aeraer asdsadas eerw dssdfsselleds
Causes of primary hyperparathyroidism
• 80%: solitary adenomahe

• 15%: hyperplasiahe
• 4%: multiple adenomahe

• 1%: carcinomahe
he earaer aeraer asdsadas eerw dssdfsselleds
Features - 'bones, stones, abdominal groans and psychic moans'
• polydipsia, polyuriahe

• peptic ulceration/constipation/pancreatitis he
• bone pain/fracturehe

• renal stoneshe
• depressionhe
• hypertensionhe

• impaired glucose tolerancehe


he earaer aeraer asdsadas eerw dssdfsselleds
Associations
• hypertensionhe
• multiple endocrine neoplasia: MEN I and IIhe
he earaer aeraer asdsadas eerw dssdfsselleds
Investigations
• raised calcium, low phosphatehe
• PTH may be raised or normalhe
• technetium-MIBI subtraction scanhe
he earaer aeraer asdsadas eerw dssdfsselleds
Treatment
total parathyroidectomyhe

Which one of the following drugs is not associated with galactorrhoea?ia

A.A Metoclopramideia
x B.A Bromocriptine
C.A Chlorpromazineia
D.A Haloperidolia
E.A Domperidoneia
End session
Bromocriptine is a treatment for galactorrhoea, rather than a cause
Prolactin and galactorrhoea
sqweqwesf erwrewfsdfs adasd dhe
Prolactin is secreted by the anterior pituitary gland with release being controlled by a
wide variety of physiological factors. Dopamine acts as the primary prolactin releasing
inhibitory factor and hence dopamine agonists such as bromocriptine may be used to
control galactorrhoea. It is important to differentiate the causes of galactorrhoea (due to
the actions of prolactin on breast tissue) from those of gynaecomastia
he earaer aeraer asdsadas eerw dssdfsselleds
Features of excess prolactin
• men: impotence, loss of libido, galactorrhoeahe
• women: amenorrhoea, galactorrhoeahe
he earaer aeraer asdsadas eerw dssdfsselleds
Causes of raised prolactin
• pregnancyhe

• oestrogenshe
• stresshe

• exercisehe
• sleephe
• acromegaly: 1/3 of patientshe
• polycystic ovarian syndromehe
• primary hypothyroidism (due to thyrotrophin releasing hormone (TRH) stimulating
prolactin release)he
he earaer aeraer asdsadas eerw dssdfsselleds
Drug causes of raised prolactin
• metoclopramide, domperidonehe
• phenothiazineshe

• haloperidolhe
• very rare: SSRIs, opioidshe
Key points from session

Addison's disease is associated with a metabolic acidosis


Bromocriptine is a treatment for galactorrhoea, rather than a cause
Causes of villous atrophy (other than coeliacs): tropical sprue, Whipple's, lymphoma,
hypogammaglobulinaemia
Deterioration in patient with hepatitis B - ? hepatocellular carcinoma
Diabetes diagnosis: fasting > 7.0, random > 11.1 - if asymptomatic need two readings
Flushing, diarrhoea, bronchospasm, tricuspid stenosis, pellagra --> carcinoid with liver
mets - diagnosis: urinary 5-HIAA
Gitelman’s syndrome: normotension with hypokalaemia
Glitazones are agonists of PPAR-gamma receptors
Insulinoma is diagnosed with supervised prolonged fasting
Patients on insulin cannot hold a HGV licence
Phaeochromocytoma: do 24 hr urinary catecholamines, not VMA etc
The diagnostic test for acromegaly is an oral glucose tolerance with growth hormone
measurements
The overnight dexamethasone suppression test is the best test to diagnosis Cushing's
syndrome
The short synacthen test is the best test to diagnose Addison's disease
bilateral idiopathic adrenal hyperplasia is the most common cause of primary
hyperaldosteronism

90

You might also like